SlideShare une entreprise Scribd logo
1  sur  135
OITE 2010

 DISEASES
Ortho Diseases

3. A 32-year-old man has had a 2-year history of
  left groin pain. Radiographs, MRI scans, and a
  biopsy specimen are seen in Figures 3a through
  3f. What is the most likely diagnosis?
1- Enchondroma
2- Chondroblastoma
3- Giant cell tumor
4- Osteonecrosis
5- Clear cell chondrosarcoma
3A




     3B
3C
3D
3E
3F
Ortho Diseases

3. A 32-year-old man has had a 2-year history of
  left groin pain. Radiographs, MRI scans, and a
  biopsy specimen are seen in Figures 3a through
  3f. What is the most likely diagnosis?
1- Enchondroma
2- Chondroblastoma
3- Giant cell tumor
4- Osteonecrosis
5- Clear cell chondrosarcoma
Clear Cell Chondrosarcoma
• Rare variant of chondrosarcoma(2% of all
  chondrosarcoma, 0.2% of biopsy-analyzed
  primary bone tumors)
• Usually low grade, slow growing, infrequent
  metastases, but frequent local recurrence
• Male:Female 1.3:1
• most commonly proximal humerus or femur
  (75%). 15% around the knee(distal femur and
  proximal tibia). In about 10%, the lesion has
  been noted in the skull, spine, ribs, pelvis,ulna or
  phalanges.
• Epiphyseal +/- extension into metaphysis.
• young adults, 20- 50 yo
• XR: expansile radiolucent bony lesion, absence of any
  periosteal reaction and absence of soft tissue mass +/-
  calcifications
• MR: Well demarcated, T1 -relatively homogeneous low
  to intermediate signal intensity, T2- heterogeneous high
  signal intensity
• Path: large round-to-oval tumor cells w/ distinct borders,
  abundant clear cytoplasm and a centrally located round
  nucleus. Typically see woven bone trabeculae within the
  microlobules or scattered between sheets of tumour
  cells. Giant cells present.
• S-100 positive
• Tx: operative resection with a wide margin.
References:
SPRINGFIELD DS, GEBHARDT MC, MCGUIRE MH: Instructional Course Lectures, The American
     Academy of Orthopaedic Surgeons - Chondrosarcoma: a Review. J. Bone Joint Surg. Am., Jan
     1996; 78: 141 - 9.
AYOUB KS, GRIMER RJ, CARTER SR, MANGHAM DC, DAVIES AM, TILLMAN RM:. Clear cell
     chondrosarcoma of bone. Sarcoma (1999) 3, 115-119.
Itälä A; Leerapun T, Inwards C, Collins M, Scully SP: An Institutional Review of Clear Cell
     Chondrosarcoma. Clin Orthopaedics and Related Research Number 440, pp. 209–212
Recommended Reading:
• Seo SW, Remotti F, Lee FYI. Chondrosarcoma
  of bone. In: Schwartz HS, ed. Orthopaedic
  Knowledge Update: Musculoskeletal Tumors 2.
  Rosemont, IL: American Academy of
  Orthopaedic Surgeons; 2007:185-195.
• Damron TA. Orthopaedic Surgery Essentials,
  Oncology and Basic Science. Philadelphia, PA:
  Lippincott Williams & Wilkins; 2008:201.
Ortho Diseases

4. When patients with rheumatoid arthritis are
  being treated with the disease-modifying
antirheumatic drug adalimumab, what is the
  optimal time for surgical intervention?
1- Immediately after infusion
2- Approximately 10 days after injection
3- Only after a year of continuous therapy
4- When drug levels are at their highest
5- When the pharmacokinetic levels reach
  equilibrium
Ortho Diseases

4. When patients with rheumatoid arthritis are
  being treated with the disease-modifying
antirheumatic drug adalimumab, what is the
  optimal time for surgical intervention?
1- Immediately after infusion
2- Approximately 10 days after injection
3- Only after a year of continuous therapy
4- When drug levels are at their highest
5- When the pharmacokinetic levels reach
  equilibrium
Adalimumab (Humira)
• Mechanism: TNF α Inhibitor
• TNFα - highly inflammatory macrophage-derived
  cytokine, plays a critical role in the joint destruction of
  patients with RA
• TNF α Inhibitors provide symptomatic and functional
  improvement and slows radiographic progression of
  disease
• Enhances the risk of infection with mycobacteria and
  other opportunistic microorganisms, as well as
  increasing risk of postoperative infections.
• Half life 10-13.6 days
References:
Giles JT, Bartlett SJ, Gelber AC, Nanda S, Fontaine K, Ruffing V, Bathon JM. Tumor necrosis factor
     inhibitor therapy and risk of serious postoperative orthopedic infection in rheumatoid arthritis.
     Arthritis Rheum. 2006 Apr 15;55(2):333-7.
Pfeiffer M, Koller M, Wnivernhaus A, Trieb K. A postoperative infectious complication in a patient with
     rheumatoid arthritis treated with adalimumab. Z Rheumatol 2006 Dec;65(8): 725-7
Recommended reading:
• Shojania K, Esdaile JM, Gredanus N. Arthritis.
  In: Vaccaro AR, ed. Orthopaedic Knowledge
  Update 8. Rosemont, IL: American Academy of
  Orthopaedic Surgeons; 2005:234-239.
• Giles JT, Bartlett SJ, Gelber AC, Nanda S,
  Fontaine K, Ruffing V, Bathon JM. Tumor
  necrosis factor inhibitor therapy and risk of
  serious postoperative orthopedic infection in
  rheumatoid arthritis. Arthritis Rheum. 2006 Apr
  15;55(2):333-7. PubMed PMID: 16583385.
Ortho Diseases

21. Which of the following is a recognized
  complication of extra-articular
  corticosteroid injections?
1- Acne
2- Depression
3- Fat atrophy
4- Hypogonadism
5- Manic episode
Ortho Diseases

21. Which of the following is a recognized
  complication of extra-articular
  corticosteroid injections?
1- Acne
2- Depression
3- Fat atrophy
4- Hypogonadism
5- Manic episode
Fat Atrophy
• More common with less soluble agents
  (eg. Triamcinilone)
• Effects can last >6mo
• Other side effects steroid injections: Post-
  injectional flare (1-10%), facial flushing
  (15%, women>men), infection, sepsis,
  skin depigmentation, perilymphatic
  atrophy, hyperpigmentation,tendon
  rupture.
Recommended Reading/References:
• Cole BJ, Schumacher HR Jr. Injectable
  corticosteroids in modern practice. J Am Acad
  Orthop Surg. 2005 Jan-Feb;13(1):37-46.
  Review. PubMed PMID: 15712981.
• Kumar N, Newman RJ. Complications of intra-
  and peri-articular steroid injections. Br J Gen
  Pract. 1999 Jun;49(443):465-6. PubMed PMID:
  10562748; PubMed Central PMCID:
  PMC1313446.
Ortho Diseases

24.Figures 24a through 24c show the radiographs
  and CT scan of a 7-year-old girl who has had
  mild discomfort and a hard mass on the anterior
  tibia for 1 year. What is the most likely
  diagnosis?
1- Chondroblastoma
2- Ewing’s sarcoma
3- Ossifying fibroma
4- Aneurysmal bone cyst
5- Parosteal osteosarcoma
24A




      24B
24C
Ortho Diseases

24.Figures 24a through 24c show the radiographs
  and CT scan of a 7-year-old girl who has had
  mild discomfort and a hard mass on the anterior
  tibia for 1 year. What is the most likely
  diagnosis?
1- Chondroblastoma
2- Ewing’s sarcoma
3- Ossifying fibroma
4- Aneurysmal bone cyst
5- Parosteal osteosarcoma
Ossifying Fibroma
• Typically occurs in the jaw bone, only rarely in long bones where they
  have been termed Osteofibrous dysplasia (OFD)
• OFD:rare (0.2% all bone tumors), benign, self-limited fibroosseous
  lesion, usually in the diaphysis of the tibia but occasionally ipsilateral
  fibula is involved
• First two decades of life
• benign lesion that rarely progresses during childhood and never
  progresses after skeletal maturity
• P/w lower leg swelling +/- pain, anterior bowing or path fx. Often
  incidental finding
• XR: intracortical lytic lesion, well marginated and is often surrounded
  by a zone of sclerosis. Cortex may be expanded or thickened.
  Typically involves anterior cortex.
• .CT can provide more information on the amount of cortical
  destruction and may reveal occult pathologic fracture
• MRI can aid in diagnosis by identifying the lesion and assessing for
  extraosseous soft tissue or intramedullary extension. OFD generally
  contained within the cortex.
Pathoogy:
•loose, often storiform fibrous
background containing spicules
of woven bony trabeculae that
are lined by a layer of
osteoblasts.
•Osteoblastic rimming
differentiates from fibrous
dysplasia.
•Devoid of devoid of epithelial
differentiation, which
differentiates fro AD



DDX: Adamantinoma(AD), fibrous dysplasia, nonossifying fibroma, UBC,
ABC, chondromyxoid fibroma, Langerhans cell histiocytosis (ie, eosinophilic
granuloma), osteomyelitis, osteosarcoma, chondrosarcoma,
hemangioendothelioma, angiosarcoma, and metastatic carcinoma
• Consider open biopsy even if radiographically
  c/w OFD as could be early AD lesion.
• Treatment controversial
  – Traditionally tx with observation. Bracing may be
    attempted to minimize deformity and prevent fracture.
    Surgery reserved for extensive or deforming lesion, or
    path fx.
  – Some have recommended aggressive surgical
    approach with extraperiosteal resection in all cases.
    Advocates site sampling error in biopsy and theory
    that OFD could progress to AD
  References:
  Most MJ, Sim FH, Inwards CY Osteofibrous Dysplasia and Adamantinoma J. Am. Acad. Ortho. Surg., June
      2010; 18: 358 - 366.

  Schoenecker PL, Swanson K, Sheridan JJ. Ossifying fibroma of the tibia. Report of a new case and review of
     the literature. JBJS 1981; 63:483-488.
Recommended Reading:
• Pitcher JD Jr, Weber KL. Benign fibrous and
  histiocytic lesions. In: Schwartz HS, ed.
  Orthopaedic Knowledge Update:
  Musculoskeletal Tumors 2. Rosemont, IL:
  American Academy of Orthopaedic Surgeons;
  2007:121-132.
• Damron TA: Orthopaedic Surgery Essentials,
  Oncology and Basic Science. Philadelphia, PA:
  Lippincott Williams & Wilkins; 2008:156-158.
Ortho Diseases

32. A 24-year-old woman reports a 4-month history
  of knee pain. Radiographs, an MRI scan, and
  biopsy specimens are seen in Figures 32a
  through 32e. What is the most likely diagnosis?
1- Chondrosarcoma
2- Ewing’s sarcoma
3- Giant cell tumor
4- Periosteal chondroma
5- Conventional osteosarcoma
32A




      32B
32C
32D




      32E
Ortho Diseases

32. A 24-year-old woman reports a 4-month history
  of knee pain. Radiographs, an MRI scan, and
  biopsy specimens are seen in Figures 32a
  through 32e. What is the most likely diagnosis?
1- Chondrosarcoma
2- Ewing’s sarcoma
3- Giant cell tumor
4- Periosteal chondroma
5- Conventional osteosarcoma
Osteosarcoma
• Most common bone sarcoma
• Peak incident 2nd decade, assoc w/ peak growth
• Most commonly occurs at sites of rapid bone turnover
  (eg. the distal femur, prox tibia, and prox humerus). Less
  commonly occurs in the pelvis, the spine, and the
  craniofacial bones.
• p/w pain, decreased ROM, mass, warmth or erythema,
  or path fx.
• XR: aggressive bone lesion arising from the metaphysis
  distal femur, proximal tibia, or proximal humerus.
  Illdefined borders, osteoblastic and/or osteolytic features,
  and an associated soft-tissue mass.
• MRI: demonstrates extent of tumor
  invasion of the surrounding soft tissues,
  neurovascular involvement, extent of bone
  marrow replacement, and presence of
  discontinuous metastases (ie, skip
  lesions)
• Biopsy
• Path: malignant mesenchymal cells w/
  pleomorphic nuclei and osteoid
Osteosarcoma Subtypes
• Conventional: most common, 1st/2nd decade. High grade,
  originating from intramedullary cavity.
• Telangiectatic osteosarcoma: <4%. Eccentric and osteolytic
  lesion, may resemble ABC. Composed of blood filled sinusoids, with
  high signal on T2.
• Low Grade: 1-2%. 3rd/4th decade. Tumor may resemble fibrous
  dysplasia, but MRI shows cortical invasion.
• Small Cell: 1.5%. Lytic areas and variable amount os sclerosis.
  Path small, round, malignant ells w/in osteoid matrix.
• Parosteal: Arise surface, sparing intramedullary canal. Pead 3 rd
  decade. Females>males. Low grade, well=differentiated fibrowus
  stroma with osseous components.
• Periosteal: 1-2%. More aggressive that paraosteal, Sunburst
  pattern or codman triangle commonly seen. Intermed grade tumor,
  mostly cartilaginous with areas of calcification.
• High Grade: <1%. Surface tumor. High grade spindle cells, w/
  atypia & varying amounts of osteoid.
• Tx: multidisciplinary.
• Low grade or parosteal: Wide surgical resection.
• High grade lesions or mets: pre-op chemo, wide
  surgical resection, post-op chemo
• Commonly used chemo agents: doxorubicin,
  cisplatin, MTX, and ifosfamide
• Without mets, 70% survival rate
• With clinically detectable mets, 20-30% survival
  rate
Messerschmitt PT, Garcia RM, Abdul-karim FW, Greenfield EM, Getty P. Osteosarcoma. JAAOS Aug
   2009; 17:515-527
Recommended Reading:
• Damron TA. Orthopaedic Surgery Essentials,
  Oncology and Basic Science. Philadelphia, PA:
  Lippincott Williams & Wilkins; 2008:177-185.
• Hornicek FJ. Osteosarcoma of bone. In:
  Schwartz HS, ed. Orthopaedic Knowledge
  Update: Musculoskeletal Tumors 2. Rosemont,
  IL: American Academy of Orthopaedic
  Surgeons; 2007:163-174.
Ortho Diseases
45.A 13-year-old boy has had right knee pain for
  the past 4 months. Radiographs, bone scan, CT
  scan, and biopsy specimen are seen in Figures
  45a through 45e. What is the most likely
  diagnosis?
1- Osteomyelitis
2- Chondroblastoma
3- Ewing’s sarcoma
4- Eosinophilic granuloma
5- Aneurysmal bone cyst
3A
 45A




         3B



       45B
3C

45C
45D
45E
Ortho Diseases
45.A 13-year-old boy has had right knee pain for
  the past 4 months. Radiographs, bone scan, CT
  scan, and biopsy specimen are seen in Figures
  45a through 45e. What is the most likely
  diagnosis?
1- Osteomyelitis
2- Chondroblastoma
3- Ewing’s sarcoma
4- Eosinophilic granuloma
5- Aneurysmal bone cyst
Chondroblastoma
• Benign neoplasm primarily affecting patients 10-
  25 years old with a male predominance of 2:1
• Most common sites include distal femur,
  proximal tibia, proximal humerus although in
  older populations it tends to involve flat bones.
• Characteristic radiographic findings include well-
  circumscribed lesions centered in an epiphysis
  of a long bone. In children, a well circumscribed
  epiphyseal lesion crossing an open growth plate
  is virtually diagnostic. There can be a
  surrounding rim of reactive bone and 30-50%
  will have some calcification evident on
  radiograph. Usually there is no soft tissue
  involvement.
• Microscopically, there are sheets of
  chondroblasts with background of
  chondroid matrix. Calcification is present
  giving a "chicken wire" appearance.
  Multinucleated giant cells are abundant.
• In adults, differential diagnoses include
  giant cell tumors (no rim of sclerotic bone
  or calcification) and clear cell
  chondrosarcoma
• Treatment includes extended curettage,
  bone grafting and/or bone cement.
Recommended Reading
- Damron TA. Orthopaedic Surgery Essentials,
  Oncology and Basic Science. Philadelphia, PA:
  Lippincott Williams & Wilkins; 2008:135-137.
- Weber KL, O’Connor MI. Benign cartilage
  tumors. In: Schwartz HS, ed. Orthopaedic
  Knowledge Update: Musculoskeletal Tumors 2.
  Rosemont, IL: American Academy of
  Orthopaedic Surgeons; 2007:103-120.g:
Ortho Diseases

57. Optimal management of osteoporosis diagnosed after a
   hip fracture includes
1- urgent medical consultation.
2- administration of bisphosphonates with follow-up as
   needed.
3- referral to the patient’s primary care physician within a
   year of surgery.
4- initiation of vitamin D and calcium supplementation with
   follow-up as needed.
5- initial evaluation by the orthopaedic team with early
   postoperative referral to an osteoporosis clinic.
Ortho Diseases

57. Optimal management of osteoporosis diagnosed after a
   hip fracture includes
1- urgent medical consultation.
2- administration of bisphosphonates with follow-up as
   needed.
3- referral to the patient’s primary care physician within a
   year of surgery.
4- initiation of vitamin D and calcium supplementation with
   follow-up as needed.
5- initial evaluation by the orthopaedic team with early
   postoperative referral to an osteoporosis clinic.
• Hip, wrist, vertebral fractures in the elderly from low
  energy mechanisms are termed fragility fractures as the
  forces involved would not fracture healthy bone.
• Miki et al's paper in JBJS 2008 was a randomized
  control trial involving 62 patients with fragility hip
  fractures. Their mean age was 79.2. Each patient was
  assigned to either an intervention or a control group.
  Patients in the intervention group were worked up for
  osteoporosis, treated and followed closely in a
  specialized orthopaedic osteoporosis clinic. Patients in
  the control group was started on treatment and told to
  follow up with their primary care doctors.
• The paper found that patients randomized to follow
  up in the specialized orthopaedic osteoporosis clinic
  had much a much high percentage of patients on
  osteoporotic medication and getting surveillance
  studies at 6 months as compared to the control
  group.
Recommended reading:
- Miki RA, Oetgen ME, Kirk J, Insogna KL,
  Lindskog DM. Orthopaedic management
  improves the rate of early osteoporosis
  treatment after hip fracture. A randomized
  clinical trial. J Bone Joint Surg Am. 2008
  Nov;90(11):2346-53. PubMed PMID: 18978403.
- Murray MA. Bone metabolism and metabolic
  bone disease. In: Vaccaro AR, ed. Orthopaedic
  Knowledge Update 8. Rosemont, IL: American
  Academy of Orthopaedic Surgeons; 2005:187-
  196.
Ortho Diseases
67. Which of the following diagnostic
  modalities is most useful for the diagnosis
  of proximal thigh deep venous
  thrombosis?
1- D-dimer assay
2- Physical examination
3- Venous ultrasonography
4- Computed tomography
5- Impedance plethysmography
Ortho Diseases
67. Which of the following diagnostic
  modalities is most useful for the diagnosis
  of proximal thigh deep venous
  thrombosis?
1- D-dimer assay
2- Physical examination
3- Venous ultrasonography
4- Computed tomography
5- Impedance plethysmography
Deep vein thrombosis
• 80-90% occur in operated limb
• Peak incidence is 4 days after surgery and minimal after
  day 17.
• Clinical diagnosis of pain, tenderness, positive Homan
  sign, swelling, erythema, low grade fever is absent in
  50% of patients
• Venography is still considered most sensitive and
  specific test for calf and thigh DVTs but is costly,
  invasive, poses risk of contrast allergy or inducing
  thrombosis
• Ultrasound has a sensitivity of 79%, specificity of
  98% and accuracy of 97% compared to venography
  but is not as useful in the diagnosis of calf and
  pelvic thrombi
Recommended Reading/References:
- Conduah AH, Lieberman JR. Thromboembolism and
   pulmonary distress in the setting of orthopaedic surgery.
   In: Einhorn TA, O’Keefe, RJ, Buckwalter JA, eds.
   Orthopaedic Basic Science: Foundations of Clinical
   Practice, 3rd ed. Rosemont, IL: American Academy of
   Orthopaedic Surgeons; 2007:105-113.
- Haut ER, Chang DC, Pierce CA, Colantuoni E, Efron DT,
   Haider AH, Cornwell EE 3rd, Pronovost PJ. Predictors of
   posttraumatic deep vein thrombosis (DVT): hospital
   practice versus patient factors-an analysis of the
   National Trauma Data Bank (NTDB). J Trauma. 2009
   Apr;66(4):994-9; discussion 999-1001. PubMed PMID:
   19359905.
Ortho Diseases

78.
Paget’s disease of bone results from a defect in
  which of the following processes?
1- Bone resorption
2- Bone regeneration
3- Osteodifferentation
4- Matrix maturation and mineralization
5- Coupling of bone formation and resorption
Ortho Diseases

78.
Paget’s disease of bone results from a defect in
  which of the following processes?
1- Bone resorption
2- Bone regeneration
3- Osteodifferentation
4- Matrix maturation and mineralization
5- Coupling of bone formation and resorption
Paget disease
•   Disorder of unregulated bone turnover. Excessive osteoclastic
    resorption is followed by increased osteoblastic activity. An early lytic
    phase is followed by excessive bone production with cortical and
    trabecular thickening.
•   More common in people over 55 years old of Anglo-Saxon descent.
•   Radiographic findings vary based on the stage of the disease. In the
    lytic phase, the bone can take on a "blade of grass" or "flame"
    appearance at the end of the bone extending to the diaphysis. Later
    on, the bone is sclerotic with thickened cortices and trabeculae. Bone
    scans can be hot. MRI is used to differentiate between malignancy
    with the marrow in Paget disease appearing normal.
•   Paget is managed medically with NSAIDs, calcitonin and
    bisphosphonates.
•   Alkaline phosphatase levels and urine pyridinium cross-links is used
    to monitor disease activity.
•   ~1% of patients with Paget disease develop a sarcoma, usually
    osteosarcoma.
Recommended Reading/References:
- Zuscik MJ, Drissi MH, Chen D, Rosier RN. Molecular and
   cell biology in orthopaedics. In: Einhorn TA, O’Keefe RJ,
   Buckwalter JA, eds. Orthopaedic Basic Science:
   Foundations of Clinical Practice, 3rd ed. Rosemont, IL:
   American Academy of Orthopaedic Surgeons; 2007:3-
   24.
- Miller JD, McCreadie BR, Alford AI, Hankenson KD,
   Goldstein SA. Form and function of bone. In: Einhorn
   TA, O’Keefe RJ, Buckwalter JA, eds. Orthopaedic Basic
   Science: Foundations of Clinical Practice, 3rd ed.
   Rosemont, IL: American Academy of Orthopaedic
   Surgeons; 2007:129-159.
Ortho Diseases
87.
A 19-year-old college football player has had
  suprapubic pain for the past 4 months. A
  radiograph and CT scans are show in Figures
  87a through 87c. What is the most likely
  diagnosis?
1- Chronic adductor strain
2- Osteoid osteoma
3- Chronic pelvic instability
4- Stress fracture
5- Osteitis pubis
87A
32A




        32B
87B




      87C
Ortho Diseases
87.
A 19-year-old college football player has had
  suprapubic pain for the past 4 months. A
  radiograph and CT scans are show in Figures
  87a through 87c. What is the most likely
  diagnosis?
1- Chronic adductor strain
2- Osteoid osteoma
3- Chronic pelvic instability
4- Stress fracture
5- Osteitis pubis
Osteitis Pubis
• Painful, noninfectious, inflammatory condition involving
  the pubic bone, symphysis and surrouding structures.
• Associated with urological/gynecological surgery and
  athletes.
• Gradual onset of pain in the pubic region is the primary
  symptom and pain may be severe enough to limit
  ambulation.
• Radiographic changes include symmetrical bone
  resorption at the medial ends of the pubic bones,
  widening of the pubic symphysis, and rarefaction or
  sclerosis along the pubic rami.
• Usually self limiting and treatment includes rest, anti-
  inflammatory medication. If symptoms persist,
  corticosteroid injections and wedge resection of the
  symphysis pubis may be necessary.
Recommended Reading/References:
- Sekiya JK, Gibbs AE. Groin and pelvis injuries:
   In: Kibler WB, ed. Orthopaedic Knowledge
   Update: Sports Medicine 4. Rosemont, IL:
   American Academy of Orthopaedic Surgeons;
   2009:83-90.
- Nuccion SL, Hunter DM, Finerman GAM. Hip and
   pelvis: adult. In: DeLee JC, Drez D Jr, Miller MD,
   eds. DeLee and Drez’s Orthopaedic Sports
   Medicine, 2nd ed. Philadelphia, PA: WB
   Saunders; 2003:1448.
Question 104
Paget’s disease is best characterized as increased bone
turnover that results:
1- from a sequenced missense genetic mutation.
2- from an autosomal-recessive inheritance pattern.
3- in significant deformity but is rarely painful.
4- in subtle radiographic findings that necessitate
advanced imaging.
5- in increased urinary N-telopeptide and alpha-C-
telopeptide.
Question 104
Paget’s disease is best characterized as increased bone
turnover that results:

1- from a sequenced missense genetic mutation.
2- from an autosomal-recessive inheritance pattern.
3- in significant deformity but is rarely painful.
4- in subtle radiographic findings that necessitate
advanced imaging.
5- in increased urinary N-telopeptide and alpha-C-
telopeptide.
Question 104
•   Paget’s disease - common geriatric problem, occurring in 3% to 4% of the
    population over 50 years of age

•   Increased bone resorption leads to a compensatory increase in bone
    formation. The overall rate of bone remodeling is accelerated, resulting in a
    predominance of highly vascular bone that is structurally weak and prone to
    deformities and pathologic fractures

•   The measurement of serum alkaline phosphatase activity is a good initial
    biochemical test for Paget’s disease. But, can be elevated in other diseases
    as well

•   Most sensitive test is urinary N-telopeptide and alpha-C-telopeptide.
Question 104
Type I Collagen
                  - Metabolic bone markers
                  relating type I collagen
                  degradation products
                  provide important
                  information on Paget’s

                  - urinary N-telopeptide
                  (NTX) and alpha-C-
                  telopeptide (CTX) for bone
                  resorption have emerged
                  as the most sensitive ones
References
Murray MA. Bone metabolism and metabolic bone disease. In: Vaccaro AR, ed.
Orthopaedic Knowledge Update 8. Rosemont, IL: American Academy of
Orthopaedic Surgeons; 2005:187- 196.

Delmas PD. Biochemical markers of bone turnover in Paget’s disease of bone.
J Bone Miner Res. 1999 Oct;14 Suppl 2:66-9. Review. PubMed PMID:
10510216.
Question 112
A 7-year-old girl has activity-related leg pain.
Radiographs and biopsy specimens are seen in
Figures 112a through 112d. What is the most likely
diagnosis?

1- Unicameral bone cyst
2- Nonossifying fibroma
3- Osteofibrous dysplasia
4- Giant cell tumor
5- Eosinophilic granuloma
Question 112
A 7-year-old girl has activity-related leg pain.
Radiographs and biopsy specimens are seen in
Figures 112a through 112d. What is the most likely
diagnosis?

1- Unicameral bone cyst
2- Nonossifying fibroma
3- Osteofibrous dysplasia
4- Giant cell tumor
5- Eosinophilic granuloma
Question 112
•   A few fun facts about                                      There is no intralesional
                                       Most commonly
    nonossifying fibromas:                                     matrix mineralization, but
                                       located in the distal   nonossifying fibromas can
     – Most common fibrous lesion      femoral metaphysis,     have an internal “bubbly”
        in children and adolescents    followed by the         appearance that is created
                                       proximal and distal     by thin, white septae
     – Radiolucent, eccentric,         tibial metaphyses.      resulting from ridging of the
        cortically based lesions                               cortex
     – Most undergo spontaneous
        regression, usually starting
        at the end of adolescence
        and disappear by age 20 to
        25 years.



        Distinct, sclerotic margins
        often with scalloping
References
•   Pitcher JD Jr, Weber KL. Benign fibrous and histiocytic lesions. In: Schwartz
    HS, ed. Orthopaedic Knowledge Update: Musculoskeletal Tumors 2.
    Rosemont, IL: American Academy of Orthopaedic Surgeons; 2007:121-132.
•   Damron TA. Orthopaedic Surgery Essentials, Oncology and Basic Science.
    Philadelphia, PA: Lippincott Williams & Wilkins; 2008:149-151.
Question 131
Patients with multiple hereditary exostosis have a greater
risk of the development of what kind of mesenchymal
tumor?

1- Hemangioma
2- Enchondroma
3- Chondrosarcoma
4- Chondroblastoma
5- Extra-abdominal desmoid
Question 131
Patients with multiple hereditary exostosis have a greater
risk of the development of what kind of mesenchymal
tumor?

1- Hemangioma
2- Enchondroma
3- Chondrosarcoma
4- Chondroblastoma
5- Extra-abdominal desmoid
Question 131
•   Hereditary multiple exostoses (HME)
     –   an autosomal dominant condition,
         mutation of tumor suppressor genes
         EXT1 or EXT2
     –   prevalence 1/50,000

•   Enlargement of exostoses rarely
    continues beyond skeletal maturity

•   Presence of an enlarging mass in an
    adult patient with HME should raise
    concerns about malignant
    transformation

•   5% of cases go on to malignant
    transformation
     –   Most commonly chondrosarcoma
     –   Lesions arising from the pelvic and
         shoulder girdles are more likely to
         undergo sarcomatous transformation    Erlenmeyer flask deformity of HME
Question 131
•   Weber KL, O’Connor MI. Benign cartilage tumors. In: Schwartz HS, ed.
    Orthopaedic Knowledge Update: Musculoskeletal Tumors 2. Rosemont, IL:
    American Academy of Orthopaedic Surgeons; 2007:103-120.
•   Damron TA. Orthopaedic Surgery Essentials, Oncology and Basic Science.
    Philadelphia, PA: Lippincott Williams & Wilkins; 2008:133.
Question 150
A 60-year-old patient who has diabetes mellitus with
peripheral neuropathy has a plantar ulcer on the plantar
surface of the second metatarsophalangeal joint.
Examination reveals no fluctuance or purulent drainage,
pulses are palpable, and the ulcer does not involve bone.
Range of motion of the ankle reveals dorsiflexion to neutral
and 15° of plantar flexion. Which of the following treatments
offers the lowest recurrence rate of ulceration?

1- Early débridement and excision of the metatarsal head
2- Second ray resection
3- IV antibiotics with ulcer débridement
4- Total contact casting
5- Total contact casting with gastrocnemius recession
Question 150
A 60-year-old patient who has diabetes mellitus with
peripheral neuropathy has a plantar ulcer on the plantar
surface of the second metatarsophalangeal joint.
Examination reveals no fluctuance or purulent drainage,
pulses are palpable, and the ulcer does not involve bone.
Range of motion of the ankle reveals dorsiflexion to neutral
and 15° of plantar flexion. Which of the following treatments
offers the lowest recurrence rate of ulceration?

1- Early débridement and excision of the metatarsal head
2- Second ray resection
3- IV antibiotics with ulcer débridement
4- Total contact casting
5- Total contact casting with gastrocnemius recession
Question 150
•   Key is recognizing that limited dorsiflexion has been implicated as a
    contributing factor to excessive plantar pressures and forefoot skin breakdown
    leading to recurrence

•   Mueller et al:
     – Randomized clinical trial involving 64 DM pts with plantar foot ulcers
     – TCC vs TCC with percutaneous Achilles lengthening


                                            TCC                   TCC with Achilles
                                                                    lengthening
          Initial healing                   88%                    100% (p>0.05)
     Recurrence < 7 months                  59%                    15% (p=0.001)
       Recurrence 2 years                   81%                    38% (p=0.002)
Question 150
•   Lin SS, Lee TH, Wapner KL. Plantar forefoot ulceration with equinus
    deformity of the ankle in diabetic patients: the effect of tendo-Achilles
    lengthening and total contact casting. Orthopedics. 1996 May;19(5):465-75.
    PubMed PMID: 8727341.
•   Mueller MJ, Sinacore DR, Hastings MK, Strube MJ, Johnson JE. Effect of
    Achilles tendon lengthening on neuropathic plantar ulcers. A randomized
    clinical trial. J Bone Joint Surg Am. 2003 Aug;85-A(8):1436-45. PubMed
    PMID: 12925622.
Question 171
Figures 171a through 171g show the radiographs, MRI
scans, and biopsy specimens of a 9-year-old girl who reports
right elbow pain that is worse at night. What is most likely
diagnosis?

1- Ewing’s sarcoma
2- Osteomyelitis
3- Giant cell tumor
4- Aneurysmal bone cyst
5- Eosinophilic granuloma
Question 171
Figures 171a through 171g show the radiographs, MRI
scans, and biopsy specimens of a 9-year-old girl who reports
right elbow pain that is worse at night. What is most likely
diagnosis?

1- Ewing’s sarcoma
2- Osteomyelitis
3- Giant cell tumor
4- Aneurysmal bone cyst
5- Eosinophilic granuloma
Question 171
•   Ewing’s Sarcoma Clinical Features:
     –   Second most common bone tumor
         (after osteosarcoma) occurring in
         children
     –   t(11;22)(q24;q12)
     –   Most common presenting symptoms of
         ES are pain, swelling, and a mass
     –   20% have fever

•   Radiographic Features on X-ray:
     –   Location: 50/50 flat bones vs. long
         bones
     –   Long bones: typically at metadiaphysis
         (58.7%) or diaphysis (35.4%)
     –   Typically, ES appears as an ill-defined,
         permeative, mottled, or focally
         motheaten, destructive intramedullary
         lesion.
     –   Periosteal reaction, with a laminated or
         “onion skin” appearance (a prominent
         multilayered reaction)
Question 171
373 patients in the
Intergroup Ewing’s Sarcoma
    Study 7299, common
    radiographic findings
Included:
1. poor margination (ie,
    indistinct lesion borders
    96%)
2. soft-tissue component
    (80.4%)
3. permeative component
    (76.1%)
4. laminated periosteal
    reaction (56.6%)
5. sclerotic component
    (39.7%)
Question 171

•   On pathology: small
    round uniform cells with
    high nucleus-
    tocytoplasmic ratio and
    grow in dense, solid
    sheets
Question 171
•   Patterson FR, Basra SK. Ewing’s sarcoma. In: Schwartz HS, ed.
    Orthopaedic Knowledge Update: Musculoskeletal Tumors 2. Rosemont, IL:
    American Academy of Orthopaedic Surgeons; 2007:175-183.
•   Damron TA. Orthopaedic Surgery Essentials, Oncology and Basic Science.
    Philadelphia, PA: Lippincott Williams & Wilkins; 2008:187-191.
204. A 26-year-old woman has had wrist pain for the
past 3 weeks. Radiographs and an MRI scan are seen in
Figures 204a through 204d. A biopsy specimen is seen
in Figure 204e. What is the most likely diagnosis?

1-   Osteosarcoma
2-   Giant cell tumor
3-   Large cell lymphoma
4-   Metasatatic carcinoma
5-   Nonossifying fibroma
204A
204B
204C
204D
204E
204. A 26-year-old woman has had wrist pain for the
past 3 weeks. Radiographs and an MRI scan are seen in
Figures 204a through 204d. A biopsy specimen is seen
in Figure 204e. What is the most likely diagnosis?

1-   Osteosarcoma
2-   Giant cell tumor
3-   Large cell lymphoma
4-   Metasatatic carcinoma
5-   Nonossifying fibroma
Giant cell tumors are commonly seen in the knee (50%), distal radius (as in this case), and
proximal humerus. They are more common in females. Radiographically, they are seen as
well circumscribed purely lytic lesions. Pathology shows sheets of multi-nucleated giant
cells.

-Osteosarcoma commonly occurs about the knee, proximal humerus, proximal femur and
pelvis of children and young adults. Radiographs show poorly defined bone forming lesions
with both bone destruction and formation. Path demonstrates lacelike mineralizing osteoid
surrounding atypical osteoblasts.
-Lymphoma is most frequently seen in the knee, pelvis, proximal femur, and vetebra, and
can occur in all ages. Lymphoma of bone is typically diffuse large B-cell type. Xrays show
poorly circumscribed lytic lesions. Path shows marrow replacement by a uniform
population of lymphoid cells.
-Metastatic disease is the most common lesion seen in older patients, particularly over the
age of 40. Most common carcinomas to metastisize to bone are breast, lung, prostate,
kidney and thyroid. Most common locations are the pelvis, vertebral bodies, ribs and
proximal limb girdles. Radiographs can show lytic, mixed, or purely sclerotic lesions.
-Nonossifying fibroma is a benign lesion most common seen in the distal femur, distal tibia,
and proximal tibia of young patients. Characteristic radiographic appearance is a lucent
lesion that is metaphyseal, eccentric, and surrounded by a sclerotic rim. Histology shows a
whorled pattern of fibroblastic cells with giant cells.
References



204.

Mirra JM. Bone Tumors. Philadelphia, PA: Lea and Febiger;
1989:941-1022.

McDonald DJ, Weber KL. Giant cell tumor of bone. In: Schwartz HS,
ed. Orthopaedic Knowledge Update: Musculoskeletal Tumors 2.
Rosemont, IL: American Academy of Orthopaedic Surgeons;
2007:133-140.
224. Figures 224a and 224b show the radiographs of a
40-year-old woman with a familial history of ‘high
arches’ with weakness and feelings of lateral ankle
instability. Significant contribution to the development
of this deformity comes from which of the following?

1-   Hindfoot arthritis
2-   Deltoid insufficiency
3-   Contracture of the gastrocsoleus complex
4-   Atrophy of the peroneus brevis muscle
5-   Weakness of the posterior tibialis muscle
224A
224B
224. Figures 224a and 224b show the radiographs of a
40-year-old woman with a familial history of ‘high
arches’ with weakness and feelings of lateral ankle
instability. Significant contribution to the development
of this deformity comes from which of the following?

1- Hindfoot arthritis
2- Deltoid insufficiency
3- Contracture of the gastrocsoleus complex
4- Atrophy of the peroneus brevis muscle 5-
Weakness of the posterior tibialis muscle
High longitudinal arches, or pes cavus, occur frequently in neurological
disorders such as Charcot-Marie-Tooth. Weakness and atrophy of the
tibialis anterior, extensor hallucis longus and the peroneus brevis are found
in pes cavus. This leads to an imbalance in the plantar and dorisiflexors, as
well as invertors and evertors of the foot and produces plantarflexion of the
first ray, hindfoot varus, and the cavus malalignment.
Patients tend to complain of painful calluses under the first and fifth
metatarsals and the medial heel.
The Coleman block test is important in determining the flexibility of the
hindfoot varus. This is important in determining treatment.
Work up should include a family history and neuro exam, as well as EMG to
evaluate for neuromuscular disorders.
Unilateral or asymmetric deformity can result from cerebral palsy, spinal
cord tumors, or a tethered cord.
Bilateral deformity can be caused by muscular dystrophy, cerebral palsy,
CMT, Friedreich’s Ataxia, Polio, or be idiopathic.
References



224.

Alexander IJ, Johnson KA. Assessment and management of pes
cavus in Charcot-Marie-Tooth disease. Clin Orthop Relat Res. 1989
Sep;(246):273-81. PubMed PMID: 2766615.

Paulos L, Coleman SS, Samuelson KM. Pes cavovarus. Review of a
surgical approach using selective soft-tissue procedures. J Bone Joint
Surg Am. 1980 Sep;62(6):942-53. PubMed PMID: 7430182.
241. Figure 241 shows the radiograph of a 40-year-old
man who reports a 1-year history of forefoot pain and
swelling. Examination reveals synovitis and pitting of the
nails. What is the most likely diagnosis?

1-   Gout
2-   Hallux rigidus
3-   Reiter’s syndrome
4-   Psoriatic arthritis
5-   Rheumatoid arthritis
241
241. Figure 241 shows the radiograph of a 40-year-old
man who reports a 1-year history of forefoot pain and
swelling. Examination reveals synovitis and pitting of the
nails. What is the most likely diagnosis?

1-   Gout
2-   Hallux rigidus
3-   Reiter’s syndrome
4-   Psoriatic arthritis
5-   Rheumatoid arthritis
Psoriatic arthritis is an seronegative spondyloarthropathy that
affects 10-30% of people with psoriasis. Classic findings
include inflammatory arthritis of distal joints, sausage digits
(dactylitis), changes in the nails (pitting, onycholysis,
keratosis), pain the in the feet and ankles, and pain in the
sacrum. Periarticular erosions can lead to the classic “pencil-
in-cup” sign on xrays.

-Gout tends to affect the 1st MTP joint (50-75% initial cases).
Characteristic radiographic signs are inordinate soft tissue
enlargement and bony erosions on both sides of the joint.
-Hallux rigidus, or stiff big toe, is a degenerative condition that
affects the MTP joint.
-Reiter’s Syndrome (Reactive arthritis) is an autoimmune
phenomena that can develop after bacterial infection
(Chlamydia, Salmonella, Shigella, etc). Characterized by triad
of inflammatory arthritis of large joints, inflammation of the
eyes (conjuctivitis or uveitis) and urethritis (“can’t pee, can’t
see, can’t climb a tree”).
-RA affects the forefoot > midfoot/hindfoot. Also tends to
predominantly affect the MTP joint.
References


241.

Berberian WS, Najarian RG. Midfoot and forefoot arthritis and
hallux rigidus. In: Pinzur MS ed. Orthopaedic Knowledge Update:
Foot and Ankle 4. Rosemont, IL; American Academy of
Orthopaedic Surgeons; 2008:201-214.

Shurnas PS, Coughlin MJ. Arthritic conditions of the foot. In:
Coughlin MJ, Mann RA, Saltzman CL, eds. Surgery of the Foot and
Ankle, 8th ed. Philadelphia, PA: Mosby Elsevier; 2007: 856-863.
253. A 43-year-old woman who is right-hand dominant fell
onto her outstretched arm while rollerblading 1 day ago.
She reports a painful wrist. Examination reveals swelling and
tenderness dorsally. Radiographs reveal a nondisplaced
transverse fracture of the distal radius. She is placed in a
short arm cast. What can be done to reduce the risk of type
1 complex regional pain syndrome?
1-   Transcutaneous electrical nerve stimulation
2-   Occupational therapy treatment for finger dexterity
3-   Strict elevation above the heart for 72 hours
4-   Alpha adrenergic blockers for 2 weeks after injury
5-   Daily oral vitamin C for 2 months
253. A 43-year-old woman who is right-hand dominant fell
onto her outstretched arm while rollerblading 1 day ago.
She reports a painful wrist. Examination reveals swelling and
tenderness dorsally. Radiographs reveal a nondisplaced
transverse fracture of the distal radius. She is placed in a
short arm cast. What can be done to reduce the risk of type
1 complex regional pain syndrome?

1-   Transcutaneous electrical nerve stimulation
2-   Occupational therapy treatment for finger dexterity
3-   Strict elevation above the heart for 72 hours
4-   Alpha adrenergic blockers for 2 weeks after injury
5-   Daily oral vitamin C for 2 months
Type 1 complex regional pain syndrome (CRPS), formerly known as reflex sympathetic
dystrophy, is characterized by pain, swelling and changes in the skin in the absence of an
identifiable nerve lesion. Type II, or causalgia, by comparison is a result of a nerve injury.
Classically, patients are thought to progress through three stages of disease (although they
may be more accurately thought of as three distinct types of the disease):
1 - severe burning pain, redness, warmth, hyperhydrosis, muscle spasm, rapid hair and nail
growth, and vasospasm
2 - intense pain, hard edema, muscle and skin atrophy
3 - irreversible skin changes (cool, glossy, dry skin), unyielding pain, marked muscle atrophy
and stiffness/joint contractures

Incidence of CRPS is higher in women, upper extremity>lower extremity. There is no
correlation with age. Common in distal radius fractures

Vitamin C has been been shown to have a protective effect against CRPS. A randomized
clinical trial published in JBJS in 2007 showed a decrease in the incidence of complex regional
pain syndrome following distal radius fracture, with a recommended dosing of 500mg daily
for 50 days. Across the treatment dosages, there was a prevalence of CRPS in the treatment
group of 2.4% (8 of 328 patients) vs. 10.1% (10 of 99 patient) of those given placebo
(p=0.002).

Treatments for CRPS include:
NSAIDs, centrally acting analgesics (cymbalta, neurontin, amitryptilene, etc), vasodilators,
steroids, transcutaneous electrical stimulation, neurolysis. Treatments are typically more
effective the earlier in the disease process they are initiated.
References

253.

Zollinger PE, Tuinebreijer WE, Breederveld RS, Kreis RW. Can
vitamin C prevent complex regional pain syndrome in patients with
wrist fractures? A randomized, controlled, multicenter dose-response
study. J Bone Joint Surg Am. 2007 Jul;89(7):1424-31. PubMed
PMID: 17606778.

Zollinger PE, Tuinebreijer WE, Kreis RW, Breederveld RS. Effect of
vitamin C on frequency of reflex sympathetic dystrophy in wrist
fractures: a randomised trial. Lancet. 1999 Dec 11;354(9195):2025-8.
PubMed PMID: 10636366.
263. The antirheumatic drug anakinra’s mode of action is
via inhibition of which of the following?

1-   IL-1
2-   IL-6
3-   Rheumatoid factor
4-   Leukocyte production
5-   TNF-α
263. The antirheumatic drug anakinra’s mode of action is
via inhibition of which of the following?

1-   IL-1
2-   IL-6
3-   Rheumatoid factor
4-   Leukocyte production
5-   TNF-α
Anakinra (brand name Kineret) is a competitive inhibitor of the
interleukin-1 receptor. It is a approved for treatment of RA in
patients with moderate or severe disease who have had an
inadequate response to other disease-modifying antirheumatic
drugs (DMARDs). By blocking IL-1, it prevents the inflammatory
and immunologic effects of this mediator.
It is administered as a subcutaneous injection daily, typically for 24
weeks. It can not be co-administered with TNF-a inhibitors, and
no head-to-head studies comparing the two have been performed.
Can be given in with methotrexate.
Common side effects include infections (40%, 2% severe), nausea,
diarrhea and GI upset, pain and erythema at injection site (70%),
and a decrease in neutrophil count (8%).
References


263.

Shojania K, Esdaile JM, Greidanus N. Arthritis. In: Vaccaro AR, ed.
Orthopaedic Knowledge Update 8. Rosemont, IL: American
Academy of Orthopaedic Surgeons; 2005:234-239.

Kalliolias GD, Liossis SN. The future of the IL-1 receptor antagonist
anakinra: from rheumatoid arthritis to adult-onset Still’s disease and
systemic-onset juvenile idiopathic arthritis. Expert Opin Investig
Drugs. 2008 Mar;17(3):349-59. Review. PubMed PMID: 18321234.
275. A 10-year-old girl has a painful right tibial tubercle.
She has swelling but no fever. Figures 275a through
275e show AP and lateral radiographs, CT scans, and an
MRI scan. Figure 275f shows a biopsy specimen. What
is the most likely diagnosis?

1-   Healing fracture
2-   Adamantinoma
3-   Ewing’s sarcoma
4-   Chondroblastoma
5-   Eosinophilic granuloma
275A
275B
275C
275D
275E
275F
275. A 10-year-old girl has a painful right tibial tubercle.
She has swelling but no fever. Figures 275a through
275e show AP and lateral radiographs, CT scans, and an
MRI scan. Figure 275f shows a biopsy specimen. What
is the most likely diagnosis?

1-   Healing fracture
2-   Adamantinoma
3-   Ewing’s sarcoma
4-   Chondroblastoma
5-   Eosinophilic granuloma
Eosinophilic granuloma is a form of Langerhans Cell Histiocysosis (Histiocytosis X). It is a
condition that mimics primary bone neoplasm. Presents with pain and swelling and any bone
can be involved. Most frequently, a single bone is involved, but multiple bones can be affected.
Radiographs will show a highly destructive lesion with well defined margins. Histology shows
characteristic proliferating eosinophilic Langerhans cells, as well as multiple bilobed
eosinophils with bright eosinophilic cytoplasm. It is commonly a self-limiting process, although
low dose radiation, or curettage with bone grafting when articular surface is at risk or
fracture is imminent are options.

-Adamantinoma is a rare neoplasm of young adults found most frequently in the tibia, but the
fibula, femur, ulna and radius are infrequently involved. Radiographs shows multiple “bubbly”
sharply circumscribed lucent defects of different sizes, with sclerotic bone between.
-Ewing’s Sarcoma is a small round cell sarcoma of children and young adults. Most commonly
found in pelvis, distal femur, proximal tibia, femoral diaphysis, and proximal humerus. Xrays
demontrate a large destructive lesion that may be purely lytic or have variable amounts of
reactive new bone formation. May have characteristic “onion skin” appearance caused by
periosteum lifted off in multiple layers.
- Chondroblastoma is a benign cartilage tumor found in young patients with open physes.
Most common locations are the distal femur, proximal tibia and proximal humerus.
Radiographs show a central region of bone destruction separated from the medullary cavity
by a rim of sclerotic bone. Histo shows chondroblasts and scattered multinucleated giant
cells.
References



275.

Mirra JM. Bone Tumors. Philadelphia, PA: Lea and Febiger;
1989:1022-1045.

Pitcher JD, Weber KL. Benign fibrous and histiocytic lesions. In:
Schwartz HS, ed. Orthopaedic Knowledge Update: Musculoskeletal
Tumors 2. Rosemont, IL: American Academy of Orthopaedic
Surgeons; 2007:121-132.

Contenu connexe

Tendances

Diagnostic Imaging of Paget's Disease
Diagnostic Imaging of Paget's DiseaseDiagnostic Imaging of Paget's Disease
Diagnostic Imaging of Paget's DiseaseMohamed M.A. Zaitoun
 
An Atlas of Musculoskeletal Oncology: Volume 4
An Atlas of Musculoskeletal Oncology: Volume 4An Atlas of Musculoskeletal Oncology: Volume 4
An Atlas of Musculoskeletal Oncology: Volume 4Amber Caldwell
 
Avascular necrosis of hip
Avascular necrosis of hipAvascular necrosis of hip
Avascular necrosis of hipvinod naneria
 
Avascular Necrosis of the Femoral Head
Avascular Necrosis of the Femoral HeadAvascular Necrosis of the Femoral Head
Avascular Necrosis of the Femoral HeadQazi Manaan
 
An Atlas of Musculoskeletal Oncology: Volume 3
An Atlas of Musculoskeletal Oncology: Volume 3An Atlas of Musculoskeletal Oncology: Volume 3
An Atlas of Musculoskeletal Oncology: Volume 3Amber Caldwell
 
Stress fracture: Causes, Investigation, Diagnosis, Treatment
Stress fracture: Causes, Investigation, Diagnosis, TreatmentStress fracture: Causes, Investigation, Diagnosis, Treatment
Stress fracture: Causes, Investigation, Diagnosis, TreatmentDr. Anurag Mittal
 
Multiple atraumatic osteoporotic vertebral fractures: Unusual cause of pain i...
Multiple atraumatic osteoporotic vertebral fractures: Unusual cause of pain i...Multiple atraumatic osteoporotic vertebral fractures: Unusual cause of pain i...
Multiple atraumatic osteoporotic vertebral fractures: Unusual cause of pain i...Apollo Hospitals
 
Bone mineral density (bmd) test
Bone mineral density (bmd) testBone mineral density (bmd) test
Bone mineral density (bmd) testapoorvaerukulla
 
Delayed Union & Nonunion of Fractures
Delayed Union & Nonunion of FracturesDelayed Union & Nonunion of Fractures
Delayed Union & Nonunion of FracturesDr. Armaan Singh
 
Biological options in avn
Biological options in avnBiological options in avn
Biological options in avnPaudel Sushil
 
Femoroacetabular Impingment: Evidence Based Tratment
Femoroacetabular Impingment: Evidence Based TratmentFemoroacetabular Impingment: Evidence Based Tratment
Femoroacetabular Impingment: Evidence Based TratmentPhysical Therapy Central
 
Infected nonunion tibia
Infected  nonunion tibiaInfected  nonunion tibia
Infected nonunion tibiaanand mishra
 
Management of femoral head osteonecrosis
Management of femoral head osteonecrosisManagement of femoral head osteonecrosis
Management of femoral head osteonecrosisSiddhartha Naru
 

Tendances (20)

Diagnostic Imaging of Paget's Disease
Diagnostic Imaging of Paget's DiseaseDiagnostic Imaging of Paget's Disease
Diagnostic Imaging of Paget's Disease
 
Vol 5 ppt
Vol 5 pptVol 5 ppt
Vol 5 ppt
 
Osteoporosis
OsteoporosisOsteoporosis
Osteoporosis
 
An Atlas of Musculoskeletal Oncology: Volume 4
An Atlas of Musculoskeletal Oncology: Volume 4An Atlas of Musculoskeletal Oncology: Volume 4
An Atlas of Musculoskeletal Oncology: Volume 4
 
Neck of Femur
Neck of FemurNeck of Femur
Neck of Femur
 
Avascular necrosis of hip
Avascular necrosis of hipAvascular necrosis of hip
Avascular necrosis of hip
 
Avascular Necrosis of the Femoral Head
Avascular Necrosis of the Femoral HeadAvascular Necrosis of the Femoral Head
Avascular Necrosis of the Femoral Head
 
An Atlas of Musculoskeletal Oncology: Volume 3
An Atlas of Musculoskeletal Oncology: Volume 3An Atlas of Musculoskeletal Oncology: Volume 3
An Atlas of Musculoskeletal Oncology: Volume 3
 
NONUNION.pptx
NONUNION.pptxNONUNION.pptx
NONUNION.pptx
 
Stress fracture: Causes, Investigation, Diagnosis, Treatment
Stress fracture: Causes, Investigation, Diagnosis, TreatmentStress fracture: Causes, Investigation, Diagnosis, Treatment
Stress fracture: Causes, Investigation, Diagnosis, Treatment
 
Multiple atraumatic osteoporotic vertebral fractures: Unusual cause of pain i...
Multiple atraumatic osteoporotic vertebral fractures: Unusual cause of pain i...Multiple atraumatic osteoporotic vertebral fractures: Unusual cause of pain i...
Multiple atraumatic osteoporotic vertebral fractures: Unusual cause of pain i...
 
Bone mineral density (bmd) test
Bone mineral density (bmd) testBone mineral density (bmd) test
Bone mineral density (bmd) test
 
Delayed Union & Nonunion of Fractures
Delayed Union & Nonunion of FracturesDelayed Union & Nonunion of Fractures
Delayed Union & Nonunion of Fractures
 
Biological options in avn
Biological options in avnBiological options in avn
Biological options in avn
 
Osteoporosis
OsteoporosisOsteoporosis
Osteoporosis
 
Paget disease
Paget diseasePaget disease
Paget disease
 
Femoroacetabular Impingment: Evidence Based Tratment
Femoroacetabular Impingment: Evidence Based TratmentFemoroacetabular Impingment: Evidence Based Tratment
Femoroacetabular Impingment: Evidence Based Tratment
 
Infected nonunion tibia
Infected  nonunion tibiaInfected  nonunion tibia
Infected nonunion tibia
 
Perthes disease
Perthes diseasePerthes disease
Perthes disease
 
Management of femoral head osteonecrosis
Management of femoral head osteonecrosisManagement of femoral head osteonecrosis
Management of femoral head osteonecrosis
 

En vedette

Lesser toe disorders - Derek Park
Lesser toe disorders - Derek ParkLesser toe disorders - Derek Park
Lesser toe disorders - Derek ParkDerek Park
 
Hallux valgus - Derek Park
Hallux valgus - Derek ParkHallux valgus - Derek Park
Hallux valgus - Derek ParkDerek Park
 
Heel pain - Derek Park
Heel pain - Derek ParkHeel pain - Derek Park
Heel pain - Derek ParkDerek Park
 
Foot and ankle trauma - Derek Park
Foot and ankle trauma - Derek ParkFoot and ankle trauma - Derek Park
Foot and ankle trauma - Derek ParkDerek Park
 
radiological signs of in orthopaedics
radiological signs of in orthopaedicsradiological signs of in orthopaedics
radiological signs of in orthopaedicssguruprasad311286
 
2009 oite review
2009 oite review2009 oite review
2009 oite reviewssweet3237
 
Using Visualizations for Music Discovery
Using Visualizations for Music DiscoveryUsing Visualizations for Music Discovery
Using Visualizations for Music DiscoveryPaul Lamere
 
Oncology oite-review-2012
Oncology oite-review-2012Oncology oite-review-2012
Oncology oite-review-2012mghbonephone
 
Orthopaedic Trauma - The Basics
Orthopaedic Trauma - The BasicsOrthopaedic Trauma - The Basics
Orthopaedic Trauma - The BasicsHiren Divecha
 
The Internet Presentation
The Internet Presentation The Internet Presentation
The Internet Presentation guest9e3d59
 

En vedette (14)

Bennet
BennetBennet
Bennet
 
Lesser toe disorders - Derek Park
Lesser toe disorders - Derek ParkLesser toe disorders - Derek Park
Lesser toe disorders - Derek Park
 
Hallux valgus - Derek Park
Hallux valgus - Derek ParkHallux valgus - Derek Park
Hallux valgus - Derek Park
 
Heel pain - Derek Park
Heel pain - Derek ParkHeel pain - Derek Park
Heel pain - Derek Park
 
Foot and ankle trauma - Derek Park
Foot and ankle trauma - Derek ParkFoot and ankle trauma - Derek Park
Foot and ankle trauma - Derek Park
 
radiological signs of in orthopaedics
radiological signs of in orthopaedicsradiological signs of in orthopaedics
radiological signs of in orthopaedics
 
Fractures
FracturesFractures
Fractures
 
Foot and heel pain
Foot and heel painFoot and heel pain
Foot and heel pain
 
2009 oite review
2009 oite review2009 oite review
2009 oite review
 
Using Visualizations for Music Discovery
Using Visualizations for Music DiscoveryUsing Visualizations for Music Discovery
Using Visualizations for Music Discovery
 
Oncology oite-review-2012
Oncology oite-review-2012Oncology oite-review-2012
Oncology oite-review-2012
 
Pelvic trauma
Pelvic traumaPelvic trauma
Pelvic trauma
 
Orthopaedic Trauma - The Basics
Orthopaedic Trauma - The BasicsOrthopaedic Trauma - The Basics
Orthopaedic Trauma - The Basics
 
The Internet Presentation
The Internet Presentation The Internet Presentation
The Internet Presentation
 

Similaire à Oite 2010 disease

Lecture metastatic breast carcinoma to the spine (final version)
Lecture metastatic breast carcinoma to the spine (final version)Lecture metastatic breast carcinoma to the spine (final version)
Lecture metastatic breast carcinoma to the spine (final version)Spiro Antoniades
 
Osteoporosis_Women's_Health_6.ppt
Osteoporosis_Women's_Health_6.pptOsteoporosis_Women's_Health_6.ppt
Osteoporosis_Women's_Health_6.pptSpesialistulangAnak
 
Metastatic bone disease
Metastatic bone disease Metastatic bone disease
Metastatic bone disease marcell wijaya
 
Fracture osteochondroma
Fracture osteochondromaFracture osteochondroma
Fracture osteochondromaDrSuresh Babu
 
Rheumatic pain indramayu 29june2013
Rheumatic pain indramayu 29june2013Rheumatic pain indramayu 29june2013
Rheumatic pain indramayu 29june2013dhoan Evridho
 
Osteoarticular tuberculosis
Osteoarticular tuberculosisOsteoarticular tuberculosis
Osteoarticular tuberculosisAmitKumarSahu31
 
Intramedullary Diffuse Astrocytoma of The Spinal Cord in a 3-Year-Old Patient...
Intramedullary Diffuse Astrocytoma of The Spinal Cord in a 3-Year-Old Patient...Intramedullary Diffuse Astrocytoma of The Spinal Cord in a 3-Year-Old Patient...
Intramedullary Diffuse Astrocytoma of The Spinal Cord in a 3-Year-Old Patient...CrimsonPublishersTNN
 
MRobbins_Fractures of the Sacrum Poster.pptx
MRobbins_Fractures of the Sacrum Poster.pptxMRobbins_Fractures of the Sacrum Poster.pptx
MRobbins_Fractures of the Sacrum Poster.pptxTemesgenAgegnehu1
 
LBP - Diagnostic Radiology UPR
LBP - Diagnostic Radiology UPRLBP - Diagnostic Radiology UPR
LBP - Diagnostic Radiology UPRE ML
 

Similaire à Oite 2010 disease (20)

Osteosarcoma
OsteosarcomaOsteosarcoma
Osteosarcoma
 
Lecture metastatic breast carcinoma to the spine (final version)
Lecture metastatic breast carcinoma to the spine (final version)Lecture metastatic breast carcinoma to the spine (final version)
Lecture metastatic breast carcinoma to the spine (final version)
 
Osteoarthritis of the hand
Osteoarthritis of the handOsteoarthritis of the hand
Osteoarthritis of the hand
 
Bioplasty
Bioplasty Bioplasty
Bioplasty
 
Osteoporosis_Women's_Health_6.ppt
Osteoporosis_Women's_Health_6.pptOsteoporosis_Women's_Health_6.ppt
Osteoporosis_Women's_Health_6.ppt
 
Bone tumors
Bone tumorsBone tumors
Bone tumors
 
Frontal osteoblastoma
Frontal osteoblastomaFrontal osteoblastoma
Frontal osteoblastoma
 
Metastatic bone disease
Metastatic bone disease Metastatic bone disease
Metastatic bone disease
 
Primary spine tumors
Primary spine tumorsPrimary spine tumors
Primary spine tumors
 
Knee joint Osteoarthritis
Knee joint OsteoarthritisKnee joint Osteoarthritis
Knee joint Osteoarthritis
 
osteoarthritis.pptx
osteoarthritis.pptxosteoarthritis.pptx
osteoarthritis.pptx
 
Fracture osteochondroma
Fracture osteochondromaFracture osteochondroma
Fracture osteochondroma
 
Rheumatic pain management
Rheumatic pain managementRheumatic pain management
Rheumatic pain management
 
Rheumatic pain indramayu 29june2013
Rheumatic pain indramayu 29june2013Rheumatic pain indramayu 29june2013
Rheumatic pain indramayu 29june2013
 
Bone Tumors Benign Ppt
Bone Tumors Benign PptBone Tumors Benign Ppt
Bone Tumors Benign Ppt
 
Osteoarticular tuberculosis
Osteoarticular tuberculosisOsteoarticular tuberculosis
Osteoarticular tuberculosis
 
Osteosarcoma
OsteosarcomaOsteosarcoma
Osteosarcoma
 
Intramedullary Diffuse Astrocytoma of The Spinal Cord in a 3-Year-Old Patient...
Intramedullary Diffuse Astrocytoma of The Spinal Cord in a 3-Year-Old Patient...Intramedullary Diffuse Astrocytoma of The Spinal Cord in a 3-Year-Old Patient...
Intramedullary Diffuse Astrocytoma of The Spinal Cord in a 3-Year-Old Patient...
 
MRobbins_Fractures of the Sacrum Poster.pptx
MRobbins_Fractures of the Sacrum Poster.pptxMRobbins_Fractures of the Sacrum Poster.pptx
MRobbins_Fractures of the Sacrum Poster.pptx
 
LBP - Diagnostic Radiology UPR
LBP - Diagnostic Radiology UPRLBP - Diagnostic Radiology UPR
LBP - Diagnostic Radiology UPR
 

Dernier

PNEUMOTHORAX AND ITS MANAGEMENTS.pdf
PNEUMOTHORAX   AND  ITS  MANAGEMENTS.pdfPNEUMOTHORAX   AND  ITS  MANAGEMENTS.pdf
PNEUMOTHORAX AND ITS MANAGEMENTS.pdfDolisha Warbi
 
Culture and Health Disorders Social change.pptx
Culture and Health Disorders Social change.pptxCulture and Health Disorders Social change.pptx
Culture and Health Disorders Social change.pptxDr. Dheeraj Kumar
 
Biomechanics- Shoulder Joint!!!!!!!!!!!!
Biomechanics- Shoulder Joint!!!!!!!!!!!!Biomechanics- Shoulder Joint!!!!!!!!!!!!
Biomechanics- Shoulder Joint!!!!!!!!!!!!ibtesaam huma
 
Basic principles involved in the traditional systems of medicine PDF.pdf
Basic principles involved in the traditional systems of medicine PDF.pdfBasic principles involved in the traditional systems of medicine PDF.pdf
Basic principles involved in the traditional systems of medicine PDF.pdfDivya Kanojiya
 
VarSeq 2.6.0: Advancing Pharmacogenomics and Genomic Analysis
VarSeq 2.6.0: Advancing Pharmacogenomics and Genomic AnalysisVarSeq 2.6.0: Advancing Pharmacogenomics and Genomic Analysis
VarSeq 2.6.0: Advancing Pharmacogenomics and Genomic AnalysisGolden Helix
 
Pharmaceutical Marketting: Unit-5, Pricing
Pharmaceutical Marketting: Unit-5, PricingPharmaceutical Marketting: Unit-5, Pricing
Pharmaceutical Marketting: Unit-5, PricingArunagarwal328757
 
world health day presentation ppt download
world health day presentation ppt downloadworld health day presentation ppt download
world health day presentation ppt downloadAnkitKumar311566
 
Informed Consent Empowering Healthcare Decision-Making.pptx
Informed Consent Empowering Healthcare Decision-Making.pptxInformed Consent Empowering Healthcare Decision-Making.pptx
Informed Consent Empowering Healthcare Decision-Making.pptxSasikiranMarri
 
Presentació "Real-Life VR Integration for Mild Cognitive Impairment Rehabilit...
Presentació "Real-Life VR Integration for Mild Cognitive Impairment Rehabilit...Presentació "Real-Life VR Integration for Mild Cognitive Impairment Rehabilit...
Presentació "Real-Life VR Integration for Mild Cognitive Impairment Rehabilit...Badalona Serveis Assistencials
 
April 2024 ONCOLOGY CARTOON by DR KANHU CHARAN PATRO
April 2024 ONCOLOGY CARTOON by  DR KANHU CHARAN PATROApril 2024 ONCOLOGY CARTOON by  DR KANHU CHARAN PATRO
April 2024 ONCOLOGY CARTOON by DR KANHU CHARAN PATROKanhu Charan
 
Big Data Analysis Suggests COVID Vaccination Increases Excess Mortality Of ...
Big Data Analysis Suggests COVID  Vaccination Increases Excess Mortality Of  ...Big Data Analysis Suggests COVID  Vaccination Increases Excess Mortality Of  ...
Big Data Analysis Suggests COVID Vaccination Increases Excess Mortality Of ...sdateam0
 
call girls in Connaught Place DELHI 🔝 >༒9540349809 🔝 genuine Escort Service ...
call girls in Connaught Place  DELHI 🔝 >༒9540349809 🔝 genuine Escort Service ...call girls in Connaught Place  DELHI 🔝 >༒9540349809 🔝 genuine Escort Service ...
call girls in Connaught Place DELHI 🔝 >༒9540349809 🔝 genuine Escort Service ...saminamagar
 
History and Development of Pharmacovigilence.pdf
History and Development of Pharmacovigilence.pdfHistory and Development of Pharmacovigilence.pdf
History and Development of Pharmacovigilence.pdfSasikiranMarri
 
call girls in aerocity DELHI 🔝 >༒9540349809 🔝 genuine Escort Service 🔝✔️✔️
call girls in aerocity DELHI 🔝 >༒9540349809 🔝 genuine Escort Service 🔝✔️✔️call girls in aerocity DELHI 🔝 >༒9540349809 🔝 genuine Escort Service 🔝✔️✔️
call girls in aerocity DELHI 🔝 >༒9540349809 🔝 genuine Escort Service 🔝✔️✔️saminamagar
 
LUNG TUMORS AND ITS CLASSIFICATIONS.pdf
LUNG TUMORS AND ITS  CLASSIFICATIONS.pdfLUNG TUMORS AND ITS  CLASSIFICATIONS.pdf
LUNG TUMORS AND ITS CLASSIFICATIONS.pdfDolisha Warbi
 
COVID-19 (NOVEL CORONA VIRUS DISEASE PANDEMIC ).pptx
COVID-19  (NOVEL CORONA  VIRUS DISEASE PANDEMIC ).pptxCOVID-19  (NOVEL CORONA  VIRUS DISEASE PANDEMIC ).pptx
COVID-19 (NOVEL CORONA VIRUS DISEASE PANDEMIC ).pptxBibekananda shah
 
Let's Talk About It: To Disclose or Not to Disclose?
Let's Talk About It: To Disclose or Not to Disclose?Let's Talk About It: To Disclose or Not to Disclose?
Let's Talk About It: To Disclose or Not to Disclose?bkling
 
SWD (Short wave diathermy)- Physiotherapy.ppt
SWD (Short wave diathermy)- Physiotherapy.pptSWD (Short wave diathermy)- Physiotherapy.ppt
SWD (Short wave diathermy)- Physiotherapy.pptMumux Mirani
 
Wessex Health Partners Wessex Integrated Care, Population Health, Research & ...
Wessex Health Partners Wessex Integrated Care, Population Health, Research & ...Wessex Health Partners Wessex Integrated Care, Population Health, Research & ...
Wessex Health Partners Wessex Integrated Care, Population Health, Research & ...Wessex Health Partners
 
POST NATAL EXERCISES AND ITS IMPACT.pptx
POST NATAL EXERCISES AND ITS IMPACT.pptxPOST NATAL EXERCISES AND ITS IMPACT.pptx
POST NATAL EXERCISES AND ITS IMPACT.pptxvirengeeta
 

Dernier (20)

PNEUMOTHORAX AND ITS MANAGEMENTS.pdf
PNEUMOTHORAX   AND  ITS  MANAGEMENTS.pdfPNEUMOTHORAX   AND  ITS  MANAGEMENTS.pdf
PNEUMOTHORAX AND ITS MANAGEMENTS.pdf
 
Culture and Health Disorders Social change.pptx
Culture and Health Disorders Social change.pptxCulture and Health Disorders Social change.pptx
Culture and Health Disorders Social change.pptx
 
Biomechanics- Shoulder Joint!!!!!!!!!!!!
Biomechanics- Shoulder Joint!!!!!!!!!!!!Biomechanics- Shoulder Joint!!!!!!!!!!!!
Biomechanics- Shoulder Joint!!!!!!!!!!!!
 
Basic principles involved in the traditional systems of medicine PDF.pdf
Basic principles involved in the traditional systems of medicine PDF.pdfBasic principles involved in the traditional systems of medicine PDF.pdf
Basic principles involved in the traditional systems of medicine PDF.pdf
 
VarSeq 2.6.0: Advancing Pharmacogenomics and Genomic Analysis
VarSeq 2.6.0: Advancing Pharmacogenomics and Genomic AnalysisVarSeq 2.6.0: Advancing Pharmacogenomics and Genomic Analysis
VarSeq 2.6.0: Advancing Pharmacogenomics and Genomic Analysis
 
Pharmaceutical Marketting: Unit-5, Pricing
Pharmaceutical Marketting: Unit-5, PricingPharmaceutical Marketting: Unit-5, Pricing
Pharmaceutical Marketting: Unit-5, Pricing
 
world health day presentation ppt download
world health day presentation ppt downloadworld health day presentation ppt download
world health day presentation ppt download
 
Informed Consent Empowering Healthcare Decision-Making.pptx
Informed Consent Empowering Healthcare Decision-Making.pptxInformed Consent Empowering Healthcare Decision-Making.pptx
Informed Consent Empowering Healthcare Decision-Making.pptx
 
Presentació "Real-Life VR Integration for Mild Cognitive Impairment Rehabilit...
Presentació "Real-Life VR Integration for Mild Cognitive Impairment Rehabilit...Presentació "Real-Life VR Integration for Mild Cognitive Impairment Rehabilit...
Presentació "Real-Life VR Integration for Mild Cognitive Impairment Rehabilit...
 
April 2024 ONCOLOGY CARTOON by DR KANHU CHARAN PATRO
April 2024 ONCOLOGY CARTOON by  DR KANHU CHARAN PATROApril 2024 ONCOLOGY CARTOON by  DR KANHU CHARAN PATRO
April 2024 ONCOLOGY CARTOON by DR KANHU CHARAN PATRO
 
Big Data Analysis Suggests COVID Vaccination Increases Excess Mortality Of ...
Big Data Analysis Suggests COVID  Vaccination Increases Excess Mortality Of  ...Big Data Analysis Suggests COVID  Vaccination Increases Excess Mortality Of  ...
Big Data Analysis Suggests COVID Vaccination Increases Excess Mortality Of ...
 
call girls in Connaught Place DELHI 🔝 >༒9540349809 🔝 genuine Escort Service ...
call girls in Connaught Place  DELHI 🔝 >༒9540349809 🔝 genuine Escort Service ...call girls in Connaught Place  DELHI 🔝 >༒9540349809 🔝 genuine Escort Service ...
call girls in Connaught Place DELHI 🔝 >༒9540349809 🔝 genuine Escort Service ...
 
History and Development of Pharmacovigilence.pdf
History and Development of Pharmacovigilence.pdfHistory and Development of Pharmacovigilence.pdf
History and Development of Pharmacovigilence.pdf
 
call girls in aerocity DELHI 🔝 >༒9540349809 🔝 genuine Escort Service 🔝✔️✔️
call girls in aerocity DELHI 🔝 >༒9540349809 🔝 genuine Escort Service 🔝✔️✔️call girls in aerocity DELHI 🔝 >༒9540349809 🔝 genuine Escort Service 🔝✔️✔️
call girls in aerocity DELHI 🔝 >༒9540349809 🔝 genuine Escort Service 🔝✔️✔️
 
LUNG TUMORS AND ITS CLASSIFICATIONS.pdf
LUNG TUMORS AND ITS  CLASSIFICATIONS.pdfLUNG TUMORS AND ITS  CLASSIFICATIONS.pdf
LUNG TUMORS AND ITS CLASSIFICATIONS.pdf
 
COVID-19 (NOVEL CORONA VIRUS DISEASE PANDEMIC ).pptx
COVID-19  (NOVEL CORONA  VIRUS DISEASE PANDEMIC ).pptxCOVID-19  (NOVEL CORONA  VIRUS DISEASE PANDEMIC ).pptx
COVID-19 (NOVEL CORONA VIRUS DISEASE PANDEMIC ).pptx
 
Let's Talk About It: To Disclose or Not to Disclose?
Let's Talk About It: To Disclose or Not to Disclose?Let's Talk About It: To Disclose or Not to Disclose?
Let's Talk About It: To Disclose or Not to Disclose?
 
SWD (Short wave diathermy)- Physiotherapy.ppt
SWD (Short wave diathermy)- Physiotherapy.pptSWD (Short wave diathermy)- Physiotherapy.ppt
SWD (Short wave diathermy)- Physiotherapy.ppt
 
Wessex Health Partners Wessex Integrated Care, Population Health, Research & ...
Wessex Health Partners Wessex Integrated Care, Population Health, Research & ...Wessex Health Partners Wessex Integrated Care, Population Health, Research & ...
Wessex Health Partners Wessex Integrated Care, Population Health, Research & ...
 
POST NATAL EXERCISES AND ITS IMPACT.pptx
POST NATAL EXERCISES AND ITS IMPACT.pptxPOST NATAL EXERCISES AND ITS IMPACT.pptx
POST NATAL EXERCISES AND ITS IMPACT.pptx
 

Oite 2010 disease

  • 2. Ortho Diseases 3. A 32-year-old man has had a 2-year history of left groin pain. Radiographs, MRI scans, and a biopsy specimen are seen in Figures 3a through 3f. What is the most likely diagnosis? 1- Enchondroma 2- Chondroblastoma 3- Giant cell tumor 4- Osteonecrosis 5- Clear cell chondrosarcoma
  • 3. 3A 3B
  • 4. 3C
  • 5. 3D
  • 6. 3E
  • 7. 3F
  • 8. Ortho Diseases 3. A 32-year-old man has had a 2-year history of left groin pain. Radiographs, MRI scans, and a biopsy specimen are seen in Figures 3a through 3f. What is the most likely diagnosis? 1- Enchondroma 2- Chondroblastoma 3- Giant cell tumor 4- Osteonecrosis 5- Clear cell chondrosarcoma
  • 9. Clear Cell Chondrosarcoma • Rare variant of chondrosarcoma(2% of all chondrosarcoma, 0.2% of biopsy-analyzed primary bone tumors) • Usually low grade, slow growing, infrequent metastases, but frequent local recurrence • Male:Female 1.3:1 • most commonly proximal humerus or femur (75%). 15% around the knee(distal femur and proximal tibia). In about 10%, the lesion has been noted in the skull, spine, ribs, pelvis,ulna or phalanges. • Epiphyseal +/- extension into metaphysis. • young adults, 20- 50 yo
  • 10. • XR: expansile radiolucent bony lesion, absence of any periosteal reaction and absence of soft tissue mass +/- calcifications • MR: Well demarcated, T1 -relatively homogeneous low to intermediate signal intensity, T2- heterogeneous high signal intensity • Path: large round-to-oval tumor cells w/ distinct borders, abundant clear cytoplasm and a centrally located round nucleus. Typically see woven bone trabeculae within the microlobules or scattered between sheets of tumour cells. Giant cells present. • S-100 positive • Tx: operative resection with a wide margin. References: SPRINGFIELD DS, GEBHARDT MC, MCGUIRE MH: Instructional Course Lectures, The American Academy of Orthopaedic Surgeons - Chondrosarcoma: a Review. J. Bone Joint Surg. Am., Jan 1996; 78: 141 - 9. AYOUB KS, GRIMER RJ, CARTER SR, MANGHAM DC, DAVIES AM, TILLMAN RM:. Clear cell chondrosarcoma of bone. Sarcoma (1999) 3, 115-119. Itälä A; Leerapun T, Inwards C, Collins M, Scully SP: An Institutional Review of Clear Cell Chondrosarcoma. Clin Orthopaedics and Related Research Number 440, pp. 209–212
  • 11. Recommended Reading: • Seo SW, Remotti F, Lee FYI. Chondrosarcoma of bone. In: Schwartz HS, ed. Orthopaedic Knowledge Update: Musculoskeletal Tumors 2. Rosemont, IL: American Academy of Orthopaedic Surgeons; 2007:185-195. • Damron TA. Orthopaedic Surgery Essentials, Oncology and Basic Science. Philadelphia, PA: Lippincott Williams & Wilkins; 2008:201.
  • 12. Ortho Diseases 4. When patients with rheumatoid arthritis are being treated with the disease-modifying antirheumatic drug adalimumab, what is the optimal time for surgical intervention? 1- Immediately after infusion 2- Approximately 10 days after injection 3- Only after a year of continuous therapy 4- When drug levels are at their highest 5- When the pharmacokinetic levels reach equilibrium
  • 13. Ortho Diseases 4. When patients with rheumatoid arthritis are being treated with the disease-modifying antirheumatic drug adalimumab, what is the optimal time for surgical intervention? 1- Immediately after infusion 2- Approximately 10 days after injection 3- Only after a year of continuous therapy 4- When drug levels are at their highest 5- When the pharmacokinetic levels reach equilibrium
  • 14. Adalimumab (Humira) • Mechanism: TNF α Inhibitor • TNFα - highly inflammatory macrophage-derived cytokine, plays a critical role in the joint destruction of patients with RA • TNF α Inhibitors provide symptomatic and functional improvement and slows radiographic progression of disease • Enhances the risk of infection with mycobacteria and other opportunistic microorganisms, as well as increasing risk of postoperative infections. • Half life 10-13.6 days References: Giles JT, Bartlett SJ, Gelber AC, Nanda S, Fontaine K, Ruffing V, Bathon JM. Tumor necrosis factor inhibitor therapy and risk of serious postoperative orthopedic infection in rheumatoid arthritis. Arthritis Rheum. 2006 Apr 15;55(2):333-7. Pfeiffer M, Koller M, Wnivernhaus A, Trieb K. A postoperative infectious complication in a patient with rheumatoid arthritis treated with adalimumab. Z Rheumatol 2006 Dec;65(8): 725-7
  • 15. Recommended reading: • Shojania K, Esdaile JM, Gredanus N. Arthritis. In: Vaccaro AR, ed. Orthopaedic Knowledge Update 8. Rosemont, IL: American Academy of Orthopaedic Surgeons; 2005:234-239. • Giles JT, Bartlett SJ, Gelber AC, Nanda S, Fontaine K, Ruffing V, Bathon JM. Tumor necrosis factor inhibitor therapy and risk of serious postoperative orthopedic infection in rheumatoid arthritis. Arthritis Rheum. 2006 Apr 15;55(2):333-7. PubMed PMID: 16583385.
  • 16. Ortho Diseases 21. Which of the following is a recognized complication of extra-articular corticosteroid injections? 1- Acne 2- Depression 3- Fat atrophy 4- Hypogonadism 5- Manic episode
  • 17. Ortho Diseases 21. Which of the following is a recognized complication of extra-articular corticosteroid injections? 1- Acne 2- Depression 3- Fat atrophy 4- Hypogonadism 5- Manic episode
  • 18. Fat Atrophy • More common with less soluble agents (eg. Triamcinilone) • Effects can last >6mo • Other side effects steroid injections: Post- injectional flare (1-10%), facial flushing (15%, women>men), infection, sepsis, skin depigmentation, perilymphatic atrophy, hyperpigmentation,tendon rupture.
  • 19. Recommended Reading/References: • Cole BJ, Schumacher HR Jr. Injectable corticosteroids in modern practice. J Am Acad Orthop Surg. 2005 Jan-Feb;13(1):37-46. Review. PubMed PMID: 15712981. • Kumar N, Newman RJ. Complications of intra- and peri-articular steroid injections. Br J Gen Pract. 1999 Jun;49(443):465-6. PubMed PMID: 10562748; PubMed Central PMCID: PMC1313446.
  • 20. Ortho Diseases 24.Figures 24a through 24c show the radiographs and CT scan of a 7-year-old girl who has had mild discomfort and a hard mass on the anterior tibia for 1 year. What is the most likely diagnosis? 1- Chondroblastoma 2- Ewing’s sarcoma 3- Ossifying fibroma 4- Aneurysmal bone cyst 5- Parosteal osteosarcoma
  • 21. 24A 24B
  • 22. 24C
  • 23. Ortho Diseases 24.Figures 24a through 24c show the radiographs and CT scan of a 7-year-old girl who has had mild discomfort and a hard mass on the anterior tibia for 1 year. What is the most likely diagnosis? 1- Chondroblastoma 2- Ewing’s sarcoma 3- Ossifying fibroma 4- Aneurysmal bone cyst 5- Parosteal osteosarcoma
  • 24. Ossifying Fibroma • Typically occurs in the jaw bone, only rarely in long bones where they have been termed Osteofibrous dysplasia (OFD) • OFD:rare (0.2% all bone tumors), benign, self-limited fibroosseous lesion, usually in the diaphysis of the tibia but occasionally ipsilateral fibula is involved • First two decades of life • benign lesion that rarely progresses during childhood and never progresses after skeletal maturity • P/w lower leg swelling +/- pain, anterior bowing or path fx. Often incidental finding • XR: intracortical lytic lesion, well marginated and is often surrounded by a zone of sclerosis. Cortex may be expanded or thickened. Typically involves anterior cortex. • .CT can provide more information on the amount of cortical destruction and may reveal occult pathologic fracture • MRI can aid in diagnosis by identifying the lesion and assessing for extraosseous soft tissue or intramedullary extension. OFD generally contained within the cortex.
  • 25. Pathoogy: •loose, often storiform fibrous background containing spicules of woven bony trabeculae that are lined by a layer of osteoblasts. •Osteoblastic rimming differentiates from fibrous dysplasia. •Devoid of devoid of epithelial differentiation, which differentiates fro AD DDX: Adamantinoma(AD), fibrous dysplasia, nonossifying fibroma, UBC, ABC, chondromyxoid fibroma, Langerhans cell histiocytosis (ie, eosinophilic granuloma), osteomyelitis, osteosarcoma, chondrosarcoma, hemangioendothelioma, angiosarcoma, and metastatic carcinoma
  • 26. • Consider open biopsy even if radiographically c/w OFD as could be early AD lesion. • Treatment controversial – Traditionally tx with observation. Bracing may be attempted to minimize deformity and prevent fracture. Surgery reserved for extensive or deforming lesion, or path fx. – Some have recommended aggressive surgical approach with extraperiosteal resection in all cases. Advocates site sampling error in biopsy and theory that OFD could progress to AD References: Most MJ, Sim FH, Inwards CY Osteofibrous Dysplasia and Adamantinoma J. Am. Acad. Ortho. Surg., June 2010; 18: 358 - 366. Schoenecker PL, Swanson K, Sheridan JJ. Ossifying fibroma of the tibia. Report of a new case and review of the literature. JBJS 1981; 63:483-488.
  • 27. Recommended Reading: • Pitcher JD Jr, Weber KL. Benign fibrous and histiocytic lesions. In: Schwartz HS, ed. Orthopaedic Knowledge Update: Musculoskeletal Tumors 2. Rosemont, IL: American Academy of Orthopaedic Surgeons; 2007:121-132. • Damron TA: Orthopaedic Surgery Essentials, Oncology and Basic Science. Philadelphia, PA: Lippincott Williams & Wilkins; 2008:156-158.
  • 28. Ortho Diseases 32. A 24-year-old woman reports a 4-month history of knee pain. Radiographs, an MRI scan, and biopsy specimens are seen in Figures 32a through 32e. What is the most likely diagnosis? 1- Chondrosarcoma 2- Ewing’s sarcoma 3- Giant cell tumor 4- Periosteal chondroma 5- Conventional osteosarcoma
  • 29. 32A 32B
  • 30. 32C
  • 31. 32D 32E
  • 32. Ortho Diseases 32. A 24-year-old woman reports a 4-month history of knee pain. Radiographs, an MRI scan, and biopsy specimens are seen in Figures 32a through 32e. What is the most likely diagnosis? 1- Chondrosarcoma 2- Ewing’s sarcoma 3- Giant cell tumor 4- Periosteal chondroma 5- Conventional osteosarcoma
  • 33. Osteosarcoma • Most common bone sarcoma • Peak incident 2nd decade, assoc w/ peak growth • Most commonly occurs at sites of rapid bone turnover (eg. the distal femur, prox tibia, and prox humerus). Less commonly occurs in the pelvis, the spine, and the craniofacial bones. • p/w pain, decreased ROM, mass, warmth or erythema, or path fx. • XR: aggressive bone lesion arising from the metaphysis distal femur, proximal tibia, or proximal humerus. Illdefined borders, osteoblastic and/or osteolytic features, and an associated soft-tissue mass.
  • 34. • MRI: demonstrates extent of tumor invasion of the surrounding soft tissues, neurovascular involvement, extent of bone marrow replacement, and presence of discontinuous metastases (ie, skip lesions) • Biopsy • Path: malignant mesenchymal cells w/ pleomorphic nuclei and osteoid
  • 35. Osteosarcoma Subtypes • Conventional: most common, 1st/2nd decade. High grade, originating from intramedullary cavity. • Telangiectatic osteosarcoma: <4%. Eccentric and osteolytic lesion, may resemble ABC. Composed of blood filled sinusoids, with high signal on T2. • Low Grade: 1-2%. 3rd/4th decade. Tumor may resemble fibrous dysplasia, but MRI shows cortical invasion. • Small Cell: 1.5%. Lytic areas and variable amount os sclerosis. Path small, round, malignant ells w/in osteoid matrix. • Parosteal: Arise surface, sparing intramedullary canal. Pead 3 rd decade. Females>males. Low grade, well=differentiated fibrowus stroma with osseous components. • Periosteal: 1-2%. More aggressive that paraosteal, Sunburst pattern or codman triangle commonly seen. Intermed grade tumor, mostly cartilaginous with areas of calcification. • High Grade: <1%. Surface tumor. High grade spindle cells, w/ atypia & varying amounts of osteoid.
  • 36. • Tx: multidisciplinary. • Low grade or parosteal: Wide surgical resection. • High grade lesions or mets: pre-op chemo, wide surgical resection, post-op chemo • Commonly used chemo agents: doxorubicin, cisplatin, MTX, and ifosfamide • Without mets, 70% survival rate • With clinically detectable mets, 20-30% survival rate Messerschmitt PT, Garcia RM, Abdul-karim FW, Greenfield EM, Getty P. Osteosarcoma. JAAOS Aug 2009; 17:515-527
  • 37. Recommended Reading: • Damron TA. Orthopaedic Surgery Essentials, Oncology and Basic Science. Philadelphia, PA: Lippincott Williams & Wilkins; 2008:177-185. • Hornicek FJ. Osteosarcoma of bone. In: Schwartz HS, ed. Orthopaedic Knowledge Update: Musculoskeletal Tumors 2. Rosemont, IL: American Academy of Orthopaedic Surgeons; 2007:163-174.
  • 38. Ortho Diseases 45.A 13-year-old boy has had right knee pain for the past 4 months. Radiographs, bone scan, CT scan, and biopsy specimen are seen in Figures 45a through 45e. What is the most likely diagnosis? 1- Osteomyelitis 2- Chondroblastoma 3- Ewing’s sarcoma 4- Eosinophilic granuloma 5- Aneurysmal bone cyst
  • 39. 3A 45A 3B 45B
  • 41. 45D
  • 42. 45E
  • 43. Ortho Diseases 45.A 13-year-old boy has had right knee pain for the past 4 months. Radiographs, bone scan, CT scan, and biopsy specimen are seen in Figures 45a through 45e. What is the most likely diagnosis? 1- Osteomyelitis 2- Chondroblastoma 3- Ewing’s sarcoma 4- Eosinophilic granuloma 5- Aneurysmal bone cyst
  • 44. Chondroblastoma • Benign neoplasm primarily affecting patients 10- 25 years old with a male predominance of 2:1 • Most common sites include distal femur, proximal tibia, proximal humerus although in older populations it tends to involve flat bones. • Characteristic radiographic findings include well- circumscribed lesions centered in an epiphysis of a long bone. In children, a well circumscribed epiphyseal lesion crossing an open growth plate is virtually diagnostic. There can be a surrounding rim of reactive bone and 30-50% will have some calcification evident on radiograph. Usually there is no soft tissue involvement.
  • 45. • Microscopically, there are sheets of chondroblasts with background of chondroid matrix. Calcification is present giving a "chicken wire" appearance. Multinucleated giant cells are abundant. • In adults, differential diagnoses include giant cell tumors (no rim of sclerotic bone or calcification) and clear cell chondrosarcoma • Treatment includes extended curettage, bone grafting and/or bone cement.
  • 46. Recommended Reading - Damron TA. Orthopaedic Surgery Essentials, Oncology and Basic Science. Philadelphia, PA: Lippincott Williams & Wilkins; 2008:135-137. - Weber KL, O’Connor MI. Benign cartilage tumors. In: Schwartz HS, ed. Orthopaedic Knowledge Update: Musculoskeletal Tumors 2. Rosemont, IL: American Academy of Orthopaedic Surgeons; 2007:103-120.g:
  • 47. Ortho Diseases 57. Optimal management of osteoporosis diagnosed after a hip fracture includes 1- urgent medical consultation. 2- administration of bisphosphonates with follow-up as needed. 3- referral to the patient’s primary care physician within a year of surgery. 4- initiation of vitamin D and calcium supplementation with follow-up as needed. 5- initial evaluation by the orthopaedic team with early postoperative referral to an osteoporosis clinic.
  • 48. Ortho Diseases 57. Optimal management of osteoporosis diagnosed after a hip fracture includes 1- urgent medical consultation. 2- administration of bisphosphonates with follow-up as needed. 3- referral to the patient’s primary care physician within a year of surgery. 4- initiation of vitamin D and calcium supplementation with follow-up as needed. 5- initial evaluation by the orthopaedic team with early postoperative referral to an osteoporosis clinic.
  • 49. • Hip, wrist, vertebral fractures in the elderly from low energy mechanisms are termed fragility fractures as the forces involved would not fracture healthy bone. • Miki et al's paper in JBJS 2008 was a randomized control trial involving 62 patients with fragility hip fractures. Their mean age was 79.2. Each patient was assigned to either an intervention or a control group. Patients in the intervention group were worked up for osteoporosis, treated and followed closely in a specialized orthopaedic osteoporosis clinic. Patients in the control group was started on treatment and told to follow up with their primary care doctors. • The paper found that patients randomized to follow up in the specialized orthopaedic osteoporosis clinic had much a much high percentage of patients on osteoporotic medication and getting surveillance studies at 6 months as compared to the control group.
  • 50. Recommended reading: - Miki RA, Oetgen ME, Kirk J, Insogna KL, Lindskog DM. Orthopaedic management improves the rate of early osteoporosis treatment after hip fracture. A randomized clinical trial. J Bone Joint Surg Am. 2008 Nov;90(11):2346-53. PubMed PMID: 18978403. - Murray MA. Bone metabolism and metabolic bone disease. In: Vaccaro AR, ed. Orthopaedic Knowledge Update 8. Rosemont, IL: American Academy of Orthopaedic Surgeons; 2005:187- 196.
  • 51. Ortho Diseases 67. Which of the following diagnostic modalities is most useful for the diagnosis of proximal thigh deep venous thrombosis? 1- D-dimer assay 2- Physical examination 3- Venous ultrasonography 4- Computed tomography 5- Impedance plethysmography
  • 52. Ortho Diseases 67. Which of the following diagnostic modalities is most useful for the diagnosis of proximal thigh deep venous thrombosis? 1- D-dimer assay 2- Physical examination 3- Venous ultrasonography 4- Computed tomography 5- Impedance plethysmography
  • 53. Deep vein thrombosis • 80-90% occur in operated limb • Peak incidence is 4 days after surgery and minimal after day 17. • Clinical diagnosis of pain, tenderness, positive Homan sign, swelling, erythema, low grade fever is absent in 50% of patients • Venography is still considered most sensitive and specific test for calf and thigh DVTs but is costly, invasive, poses risk of contrast allergy or inducing thrombosis • Ultrasound has a sensitivity of 79%, specificity of 98% and accuracy of 97% compared to venography but is not as useful in the diagnosis of calf and pelvic thrombi
  • 54. Recommended Reading/References: - Conduah AH, Lieberman JR. Thromboembolism and pulmonary distress in the setting of orthopaedic surgery. In: Einhorn TA, O’Keefe, RJ, Buckwalter JA, eds. Orthopaedic Basic Science: Foundations of Clinical Practice, 3rd ed. Rosemont, IL: American Academy of Orthopaedic Surgeons; 2007:105-113. - Haut ER, Chang DC, Pierce CA, Colantuoni E, Efron DT, Haider AH, Cornwell EE 3rd, Pronovost PJ. Predictors of posttraumatic deep vein thrombosis (DVT): hospital practice versus patient factors-an analysis of the National Trauma Data Bank (NTDB). J Trauma. 2009 Apr;66(4):994-9; discussion 999-1001. PubMed PMID: 19359905.
  • 55. Ortho Diseases 78. Paget’s disease of bone results from a defect in which of the following processes? 1- Bone resorption 2- Bone regeneration 3- Osteodifferentation 4- Matrix maturation and mineralization 5- Coupling of bone formation and resorption
  • 56. Ortho Diseases 78. Paget’s disease of bone results from a defect in which of the following processes? 1- Bone resorption 2- Bone regeneration 3- Osteodifferentation 4- Matrix maturation and mineralization 5- Coupling of bone formation and resorption
  • 57. Paget disease • Disorder of unregulated bone turnover. Excessive osteoclastic resorption is followed by increased osteoblastic activity. An early lytic phase is followed by excessive bone production with cortical and trabecular thickening. • More common in people over 55 years old of Anglo-Saxon descent. • Radiographic findings vary based on the stage of the disease. In the lytic phase, the bone can take on a "blade of grass" or "flame" appearance at the end of the bone extending to the diaphysis. Later on, the bone is sclerotic with thickened cortices and trabeculae. Bone scans can be hot. MRI is used to differentiate between malignancy with the marrow in Paget disease appearing normal. • Paget is managed medically with NSAIDs, calcitonin and bisphosphonates. • Alkaline phosphatase levels and urine pyridinium cross-links is used to monitor disease activity. • ~1% of patients with Paget disease develop a sarcoma, usually osteosarcoma.
  • 58. Recommended Reading/References: - Zuscik MJ, Drissi MH, Chen D, Rosier RN. Molecular and cell biology in orthopaedics. In: Einhorn TA, O’Keefe RJ, Buckwalter JA, eds. Orthopaedic Basic Science: Foundations of Clinical Practice, 3rd ed. Rosemont, IL: American Academy of Orthopaedic Surgeons; 2007:3- 24. - Miller JD, McCreadie BR, Alford AI, Hankenson KD, Goldstein SA. Form and function of bone. In: Einhorn TA, O’Keefe RJ, Buckwalter JA, eds. Orthopaedic Basic Science: Foundations of Clinical Practice, 3rd ed. Rosemont, IL: American Academy of Orthopaedic Surgeons; 2007:129-159.
  • 59. Ortho Diseases 87. A 19-year-old college football player has had suprapubic pain for the past 4 months. A radiograph and CT scans are show in Figures 87a through 87c. What is the most likely diagnosis? 1- Chronic adductor strain 2- Osteoid osteoma 3- Chronic pelvic instability 4- Stress fracture 5- Osteitis pubis
  • 60. 87A 32A 32B
  • 61. 87B 87C
  • 62. Ortho Diseases 87. A 19-year-old college football player has had suprapubic pain for the past 4 months. A radiograph and CT scans are show in Figures 87a through 87c. What is the most likely diagnosis? 1- Chronic adductor strain 2- Osteoid osteoma 3- Chronic pelvic instability 4- Stress fracture 5- Osteitis pubis
  • 63. Osteitis Pubis • Painful, noninfectious, inflammatory condition involving the pubic bone, symphysis and surrouding structures. • Associated with urological/gynecological surgery and athletes. • Gradual onset of pain in the pubic region is the primary symptom and pain may be severe enough to limit ambulation. • Radiographic changes include symmetrical bone resorption at the medial ends of the pubic bones, widening of the pubic symphysis, and rarefaction or sclerosis along the pubic rami. • Usually self limiting and treatment includes rest, anti- inflammatory medication. If symptoms persist, corticosteroid injections and wedge resection of the symphysis pubis may be necessary.
  • 64. Recommended Reading/References: - Sekiya JK, Gibbs AE. Groin and pelvis injuries: In: Kibler WB, ed. Orthopaedic Knowledge Update: Sports Medicine 4. Rosemont, IL: American Academy of Orthopaedic Surgeons; 2009:83-90. - Nuccion SL, Hunter DM, Finerman GAM. Hip and pelvis: adult. In: DeLee JC, Drez D Jr, Miller MD, eds. DeLee and Drez’s Orthopaedic Sports Medicine, 2nd ed. Philadelphia, PA: WB Saunders; 2003:1448.
  • 65. Question 104 Paget’s disease is best characterized as increased bone turnover that results: 1- from a sequenced missense genetic mutation. 2- from an autosomal-recessive inheritance pattern. 3- in significant deformity but is rarely painful. 4- in subtle radiographic findings that necessitate advanced imaging. 5- in increased urinary N-telopeptide and alpha-C- telopeptide.
  • 66. Question 104 Paget’s disease is best characterized as increased bone turnover that results: 1- from a sequenced missense genetic mutation. 2- from an autosomal-recessive inheritance pattern. 3- in significant deformity but is rarely painful. 4- in subtle radiographic findings that necessitate advanced imaging. 5- in increased urinary N-telopeptide and alpha-C- telopeptide.
  • 67. Question 104 • Paget’s disease - common geriatric problem, occurring in 3% to 4% of the population over 50 years of age • Increased bone resorption leads to a compensatory increase in bone formation. The overall rate of bone remodeling is accelerated, resulting in a predominance of highly vascular bone that is structurally weak and prone to deformities and pathologic fractures • The measurement of serum alkaline phosphatase activity is a good initial biochemical test for Paget’s disease. But, can be elevated in other diseases as well • Most sensitive test is urinary N-telopeptide and alpha-C-telopeptide.
  • 68. Question 104 Type I Collagen - Metabolic bone markers relating type I collagen degradation products provide important information on Paget’s - urinary N-telopeptide (NTX) and alpha-C- telopeptide (CTX) for bone resorption have emerged as the most sensitive ones
  • 69. References Murray MA. Bone metabolism and metabolic bone disease. In: Vaccaro AR, ed. Orthopaedic Knowledge Update 8. Rosemont, IL: American Academy of Orthopaedic Surgeons; 2005:187- 196. Delmas PD. Biochemical markers of bone turnover in Paget’s disease of bone. J Bone Miner Res. 1999 Oct;14 Suppl 2:66-9. Review. PubMed PMID: 10510216.
  • 70. Question 112 A 7-year-old girl has activity-related leg pain. Radiographs and biopsy specimens are seen in Figures 112a through 112d. What is the most likely diagnosis? 1- Unicameral bone cyst 2- Nonossifying fibroma 3- Osteofibrous dysplasia 4- Giant cell tumor 5- Eosinophilic granuloma
  • 71.
  • 72.
  • 73. Question 112 A 7-year-old girl has activity-related leg pain. Radiographs and biopsy specimens are seen in Figures 112a through 112d. What is the most likely diagnosis? 1- Unicameral bone cyst 2- Nonossifying fibroma 3- Osteofibrous dysplasia 4- Giant cell tumor 5- Eosinophilic granuloma
  • 74. Question 112 • A few fun facts about There is no intralesional Most commonly nonossifying fibromas: matrix mineralization, but located in the distal nonossifying fibromas can – Most common fibrous lesion femoral metaphysis, have an internal “bubbly” in children and adolescents followed by the appearance that is created proximal and distal by thin, white septae – Radiolucent, eccentric, tibial metaphyses. resulting from ridging of the cortically based lesions cortex – Most undergo spontaneous regression, usually starting at the end of adolescence and disappear by age 20 to 25 years. Distinct, sclerotic margins often with scalloping
  • 75. References • Pitcher JD Jr, Weber KL. Benign fibrous and histiocytic lesions. In: Schwartz HS, ed. Orthopaedic Knowledge Update: Musculoskeletal Tumors 2. Rosemont, IL: American Academy of Orthopaedic Surgeons; 2007:121-132. • Damron TA. Orthopaedic Surgery Essentials, Oncology and Basic Science. Philadelphia, PA: Lippincott Williams & Wilkins; 2008:149-151.
  • 76. Question 131 Patients with multiple hereditary exostosis have a greater risk of the development of what kind of mesenchymal tumor? 1- Hemangioma 2- Enchondroma 3- Chondrosarcoma 4- Chondroblastoma 5- Extra-abdominal desmoid
  • 77. Question 131 Patients with multiple hereditary exostosis have a greater risk of the development of what kind of mesenchymal tumor? 1- Hemangioma 2- Enchondroma 3- Chondrosarcoma 4- Chondroblastoma 5- Extra-abdominal desmoid
  • 78. Question 131 • Hereditary multiple exostoses (HME) – an autosomal dominant condition, mutation of tumor suppressor genes EXT1 or EXT2 – prevalence 1/50,000 • Enlargement of exostoses rarely continues beyond skeletal maturity • Presence of an enlarging mass in an adult patient with HME should raise concerns about malignant transformation • 5% of cases go on to malignant transformation – Most commonly chondrosarcoma – Lesions arising from the pelvic and shoulder girdles are more likely to undergo sarcomatous transformation Erlenmeyer flask deformity of HME
  • 79. Question 131 • Weber KL, O’Connor MI. Benign cartilage tumors. In: Schwartz HS, ed. Orthopaedic Knowledge Update: Musculoskeletal Tumors 2. Rosemont, IL: American Academy of Orthopaedic Surgeons; 2007:103-120. • Damron TA. Orthopaedic Surgery Essentials, Oncology and Basic Science. Philadelphia, PA: Lippincott Williams & Wilkins; 2008:133.
  • 80. Question 150 A 60-year-old patient who has diabetes mellitus with peripheral neuropathy has a plantar ulcer on the plantar surface of the second metatarsophalangeal joint. Examination reveals no fluctuance or purulent drainage, pulses are palpable, and the ulcer does not involve bone. Range of motion of the ankle reveals dorsiflexion to neutral and 15° of plantar flexion. Which of the following treatments offers the lowest recurrence rate of ulceration? 1- Early débridement and excision of the metatarsal head 2- Second ray resection 3- IV antibiotics with ulcer débridement 4- Total contact casting 5- Total contact casting with gastrocnemius recession
  • 81. Question 150 A 60-year-old patient who has diabetes mellitus with peripheral neuropathy has a plantar ulcer on the plantar surface of the second metatarsophalangeal joint. Examination reveals no fluctuance or purulent drainage, pulses are palpable, and the ulcer does not involve bone. Range of motion of the ankle reveals dorsiflexion to neutral and 15° of plantar flexion. Which of the following treatments offers the lowest recurrence rate of ulceration? 1- Early débridement and excision of the metatarsal head 2- Second ray resection 3- IV antibiotics with ulcer débridement 4- Total contact casting 5- Total contact casting with gastrocnemius recession
  • 82. Question 150 • Key is recognizing that limited dorsiflexion has been implicated as a contributing factor to excessive plantar pressures and forefoot skin breakdown leading to recurrence • Mueller et al: – Randomized clinical trial involving 64 DM pts with plantar foot ulcers – TCC vs TCC with percutaneous Achilles lengthening TCC TCC with Achilles lengthening Initial healing 88% 100% (p>0.05) Recurrence < 7 months 59% 15% (p=0.001) Recurrence 2 years 81% 38% (p=0.002)
  • 83. Question 150 • Lin SS, Lee TH, Wapner KL. Plantar forefoot ulceration with equinus deformity of the ankle in diabetic patients: the effect of tendo-Achilles lengthening and total contact casting. Orthopedics. 1996 May;19(5):465-75. PubMed PMID: 8727341. • Mueller MJ, Sinacore DR, Hastings MK, Strube MJ, Johnson JE. Effect of Achilles tendon lengthening on neuropathic plantar ulcers. A randomized clinical trial. J Bone Joint Surg Am. 2003 Aug;85-A(8):1436-45. PubMed PMID: 12925622.
  • 84. Question 171 Figures 171a through 171g show the radiographs, MRI scans, and biopsy specimens of a 9-year-old girl who reports right elbow pain that is worse at night. What is most likely diagnosis? 1- Ewing’s sarcoma 2- Osteomyelitis 3- Giant cell tumor 4- Aneurysmal bone cyst 5- Eosinophilic granuloma
  • 85.
  • 86.
  • 87.
  • 88.
  • 89.
  • 90.
  • 91.
  • 92.
  • 93. Question 171 Figures 171a through 171g show the radiographs, MRI scans, and biopsy specimens of a 9-year-old girl who reports right elbow pain that is worse at night. What is most likely diagnosis? 1- Ewing’s sarcoma 2- Osteomyelitis 3- Giant cell tumor 4- Aneurysmal bone cyst 5- Eosinophilic granuloma
  • 94. Question 171 • Ewing’s Sarcoma Clinical Features: – Second most common bone tumor (after osteosarcoma) occurring in children – t(11;22)(q24;q12) – Most common presenting symptoms of ES are pain, swelling, and a mass – 20% have fever • Radiographic Features on X-ray: – Location: 50/50 flat bones vs. long bones – Long bones: typically at metadiaphysis (58.7%) or diaphysis (35.4%) – Typically, ES appears as an ill-defined, permeative, mottled, or focally motheaten, destructive intramedullary lesion. – Periosteal reaction, with a laminated or “onion skin” appearance (a prominent multilayered reaction)
  • 95. Question 171 373 patients in the Intergroup Ewing’s Sarcoma Study 7299, common radiographic findings Included: 1. poor margination (ie, indistinct lesion borders 96%) 2. soft-tissue component (80.4%) 3. permeative component (76.1%) 4. laminated periosteal reaction (56.6%) 5. sclerotic component (39.7%)
  • 96. Question 171 • On pathology: small round uniform cells with high nucleus- tocytoplasmic ratio and grow in dense, solid sheets
  • 97. Question 171 • Patterson FR, Basra SK. Ewing’s sarcoma. In: Schwartz HS, ed. Orthopaedic Knowledge Update: Musculoskeletal Tumors 2. Rosemont, IL: American Academy of Orthopaedic Surgeons; 2007:175-183. • Damron TA. Orthopaedic Surgery Essentials, Oncology and Basic Science. Philadelphia, PA: Lippincott Williams & Wilkins; 2008:187-191.
  • 98. 204. A 26-year-old woman has had wrist pain for the past 3 weeks. Radiographs and an MRI scan are seen in Figures 204a through 204d. A biopsy specimen is seen in Figure 204e. What is the most likely diagnosis? 1- Osteosarcoma 2- Giant cell tumor 3- Large cell lymphoma 4- Metasatatic carcinoma 5- Nonossifying fibroma
  • 99. 204A
  • 100. 204B
  • 101. 204C
  • 102. 204D
  • 103. 204E
  • 104. 204. A 26-year-old woman has had wrist pain for the past 3 weeks. Radiographs and an MRI scan are seen in Figures 204a through 204d. A biopsy specimen is seen in Figure 204e. What is the most likely diagnosis? 1- Osteosarcoma 2- Giant cell tumor 3- Large cell lymphoma 4- Metasatatic carcinoma 5- Nonossifying fibroma
  • 105. Giant cell tumors are commonly seen in the knee (50%), distal radius (as in this case), and proximal humerus. They are more common in females. Radiographically, they are seen as well circumscribed purely lytic lesions. Pathology shows sheets of multi-nucleated giant cells. -Osteosarcoma commonly occurs about the knee, proximal humerus, proximal femur and pelvis of children and young adults. Radiographs show poorly defined bone forming lesions with both bone destruction and formation. Path demonstrates lacelike mineralizing osteoid surrounding atypical osteoblasts. -Lymphoma is most frequently seen in the knee, pelvis, proximal femur, and vetebra, and can occur in all ages. Lymphoma of bone is typically diffuse large B-cell type. Xrays show poorly circumscribed lytic lesions. Path shows marrow replacement by a uniform population of lymphoid cells. -Metastatic disease is the most common lesion seen in older patients, particularly over the age of 40. Most common carcinomas to metastisize to bone are breast, lung, prostate, kidney and thyroid. Most common locations are the pelvis, vertebral bodies, ribs and proximal limb girdles. Radiographs can show lytic, mixed, or purely sclerotic lesions. -Nonossifying fibroma is a benign lesion most common seen in the distal femur, distal tibia, and proximal tibia of young patients. Characteristic radiographic appearance is a lucent lesion that is metaphyseal, eccentric, and surrounded by a sclerotic rim. Histology shows a whorled pattern of fibroblastic cells with giant cells.
  • 106. References 204. Mirra JM. Bone Tumors. Philadelphia, PA: Lea and Febiger; 1989:941-1022. McDonald DJ, Weber KL. Giant cell tumor of bone. In: Schwartz HS, ed. Orthopaedic Knowledge Update: Musculoskeletal Tumors 2. Rosemont, IL: American Academy of Orthopaedic Surgeons; 2007:133-140.
  • 107. 224. Figures 224a and 224b show the radiographs of a 40-year-old woman with a familial history of ‘high arches’ with weakness and feelings of lateral ankle instability. Significant contribution to the development of this deformity comes from which of the following? 1- Hindfoot arthritis 2- Deltoid insufficiency 3- Contracture of the gastrocsoleus complex 4- Atrophy of the peroneus brevis muscle 5- Weakness of the posterior tibialis muscle
  • 108. 224A
  • 109. 224B
  • 110. 224. Figures 224a and 224b show the radiographs of a 40-year-old woman with a familial history of ‘high arches’ with weakness and feelings of lateral ankle instability. Significant contribution to the development of this deformity comes from which of the following? 1- Hindfoot arthritis 2- Deltoid insufficiency 3- Contracture of the gastrocsoleus complex 4- Atrophy of the peroneus brevis muscle 5- Weakness of the posterior tibialis muscle
  • 111. High longitudinal arches, or pes cavus, occur frequently in neurological disorders such as Charcot-Marie-Tooth. Weakness and atrophy of the tibialis anterior, extensor hallucis longus and the peroneus brevis are found in pes cavus. This leads to an imbalance in the plantar and dorisiflexors, as well as invertors and evertors of the foot and produces plantarflexion of the first ray, hindfoot varus, and the cavus malalignment. Patients tend to complain of painful calluses under the first and fifth metatarsals and the medial heel. The Coleman block test is important in determining the flexibility of the hindfoot varus. This is important in determining treatment. Work up should include a family history and neuro exam, as well as EMG to evaluate for neuromuscular disorders. Unilateral or asymmetric deformity can result from cerebral palsy, spinal cord tumors, or a tethered cord. Bilateral deformity can be caused by muscular dystrophy, cerebral palsy, CMT, Friedreich’s Ataxia, Polio, or be idiopathic.
  • 112. References 224. Alexander IJ, Johnson KA. Assessment and management of pes cavus in Charcot-Marie-Tooth disease. Clin Orthop Relat Res. 1989 Sep;(246):273-81. PubMed PMID: 2766615. Paulos L, Coleman SS, Samuelson KM. Pes cavovarus. Review of a surgical approach using selective soft-tissue procedures. J Bone Joint Surg Am. 1980 Sep;62(6):942-53. PubMed PMID: 7430182.
  • 113. 241. Figure 241 shows the radiograph of a 40-year-old man who reports a 1-year history of forefoot pain and swelling. Examination reveals synovitis and pitting of the nails. What is the most likely diagnosis? 1- Gout 2- Hallux rigidus 3- Reiter’s syndrome 4- Psoriatic arthritis 5- Rheumatoid arthritis
  • 114. 241
  • 115. 241. Figure 241 shows the radiograph of a 40-year-old man who reports a 1-year history of forefoot pain and swelling. Examination reveals synovitis and pitting of the nails. What is the most likely diagnosis? 1- Gout 2- Hallux rigidus 3- Reiter’s syndrome 4- Psoriatic arthritis 5- Rheumatoid arthritis
  • 116. Psoriatic arthritis is an seronegative spondyloarthropathy that affects 10-30% of people with psoriasis. Classic findings include inflammatory arthritis of distal joints, sausage digits (dactylitis), changes in the nails (pitting, onycholysis, keratosis), pain the in the feet and ankles, and pain in the sacrum. Periarticular erosions can lead to the classic “pencil- in-cup” sign on xrays. -Gout tends to affect the 1st MTP joint (50-75% initial cases). Characteristic radiographic signs are inordinate soft tissue enlargement and bony erosions on both sides of the joint. -Hallux rigidus, or stiff big toe, is a degenerative condition that affects the MTP joint. -Reiter’s Syndrome (Reactive arthritis) is an autoimmune phenomena that can develop after bacterial infection (Chlamydia, Salmonella, Shigella, etc). Characterized by triad of inflammatory arthritis of large joints, inflammation of the eyes (conjuctivitis or uveitis) and urethritis (“can’t pee, can’t see, can’t climb a tree”). -RA affects the forefoot > midfoot/hindfoot. Also tends to predominantly affect the MTP joint.
  • 117. References 241. Berberian WS, Najarian RG. Midfoot and forefoot arthritis and hallux rigidus. In: Pinzur MS ed. Orthopaedic Knowledge Update: Foot and Ankle 4. Rosemont, IL; American Academy of Orthopaedic Surgeons; 2008:201-214. Shurnas PS, Coughlin MJ. Arthritic conditions of the foot. In: Coughlin MJ, Mann RA, Saltzman CL, eds. Surgery of the Foot and Ankle, 8th ed. Philadelphia, PA: Mosby Elsevier; 2007: 856-863.
  • 118. 253. A 43-year-old woman who is right-hand dominant fell onto her outstretched arm while rollerblading 1 day ago. She reports a painful wrist. Examination reveals swelling and tenderness dorsally. Radiographs reveal a nondisplaced transverse fracture of the distal radius. She is placed in a short arm cast. What can be done to reduce the risk of type 1 complex regional pain syndrome? 1- Transcutaneous electrical nerve stimulation 2- Occupational therapy treatment for finger dexterity 3- Strict elevation above the heart for 72 hours 4- Alpha adrenergic blockers for 2 weeks after injury 5- Daily oral vitamin C for 2 months
  • 119. 253. A 43-year-old woman who is right-hand dominant fell onto her outstretched arm while rollerblading 1 day ago. She reports a painful wrist. Examination reveals swelling and tenderness dorsally. Radiographs reveal a nondisplaced transverse fracture of the distal radius. She is placed in a short arm cast. What can be done to reduce the risk of type 1 complex regional pain syndrome? 1- Transcutaneous electrical nerve stimulation 2- Occupational therapy treatment for finger dexterity 3- Strict elevation above the heart for 72 hours 4- Alpha adrenergic blockers for 2 weeks after injury 5- Daily oral vitamin C for 2 months
  • 120. Type 1 complex regional pain syndrome (CRPS), formerly known as reflex sympathetic dystrophy, is characterized by pain, swelling and changes in the skin in the absence of an identifiable nerve lesion. Type II, or causalgia, by comparison is a result of a nerve injury. Classically, patients are thought to progress through three stages of disease (although they may be more accurately thought of as three distinct types of the disease): 1 - severe burning pain, redness, warmth, hyperhydrosis, muscle spasm, rapid hair and nail growth, and vasospasm 2 - intense pain, hard edema, muscle and skin atrophy 3 - irreversible skin changes (cool, glossy, dry skin), unyielding pain, marked muscle atrophy and stiffness/joint contractures Incidence of CRPS is higher in women, upper extremity>lower extremity. There is no correlation with age. Common in distal radius fractures Vitamin C has been been shown to have a protective effect against CRPS. A randomized clinical trial published in JBJS in 2007 showed a decrease in the incidence of complex regional pain syndrome following distal radius fracture, with a recommended dosing of 500mg daily for 50 days. Across the treatment dosages, there was a prevalence of CRPS in the treatment group of 2.4% (8 of 328 patients) vs. 10.1% (10 of 99 patient) of those given placebo (p=0.002). Treatments for CRPS include: NSAIDs, centrally acting analgesics (cymbalta, neurontin, amitryptilene, etc), vasodilators, steroids, transcutaneous electrical stimulation, neurolysis. Treatments are typically more effective the earlier in the disease process they are initiated.
  • 121. References 253. Zollinger PE, Tuinebreijer WE, Breederveld RS, Kreis RW. Can vitamin C prevent complex regional pain syndrome in patients with wrist fractures? A randomized, controlled, multicenter dose-response study. J Bone Joint Surg Am. 2007 Jul;89(7):1424-31. PubMed PMID: 17606778. Zollinger PE, Tuinebreijer WE, Kreis RW, Breederveld RS. Effect of vitamin C on frequency of reflex sympathetic dystrophy in wrist fractures: a randomised trial. Lancet. 1999 Dec 11;354(9195):2025-8. PubMed PMID: 10636366.
  • 122. 263. The antirheumatic drug anakinra’s mode of action is via inhibition of which of the following? 1- IL-1 2- IL-6 3- Rheumatoid factor 4- Leukocyte production 5- TNF-α
  • 123. 263. The antirheumatic drug anakinra’s mode of action is via inhibition of which of the following? 1- IL-1 2- IL-6 3- Rheumatoid factor 4- Leukocyte production 5- TNF-α
  • 124. Anakinra (brand name Kineret) is a competitive inhibitor of the interleukin-1 receptor. It is a approved for treatment of RA in patients with moderate or severe disease who have had an inadequate response to other disease-modifying antirheumatic drugs (DMARDs). By blocking IL-1, it prevents the inflammatory and immunologic effects of this mediator. It is administered as a subcutaneous injection daily, typically for 24 weeks. It can not be co-administered with TNF-a inhibitors, and no head-to-head studies comparing the two have been performed. Can be given in with methotrexate. Common side effects include infections (40%, 2% severe), nausea, diarrhea and GI upset, pain and erythema at injection site (70%), and a decrease in neutrophil count (8%).
  • 125. References 263. Shojania K, Esdaile JM, Greidanus N. Arthritis. In: Vaccaro AR, ed. Orthopaedic Knowledge Update 8. Rosemont, IL: American Academy of Orthopaedic Surgeons; 2005:234-239. Kalliolias GD, Liossis SN. The future of the IL-1 receptor antagonist anakinra: from rheumatoid arthritis to adult-onset Still’s disease and systemic-onset juvenile idiopathic arthritis. Expert Opin Investig Drugs. 2008 Mar;17(3):349-59. Review. PubMed PMID: 18321234.
  • 126. 275. A 10-year-old girl has a painful right tibial tubercle. She has swelling but no fever. Figures 275a through 275e show AP and lateral radiographs, CT scans, and an MRI scan. Figure 275f shows a biopsy specimen. What is the most likely diagnosis? 1- Healing fracture 2- Adamantinoma 3- Ewing’s sarcoma 4- Chondroblastoma 5- Eosinophilic granuloma
  • 127. 275A
  • 128. 275B
  • 129. 275C
  • 130. 275D
  • 131. 275E
  • 132. 275F
  • 133. 275. A 10-year-old girl has a painful right tibial tubercle. She has swelling but no fever. Figures 275a through 275e show AP and lateral radiographs, CT scans, and an MRI scan. Figure 275f shows a biopsy specimen. What is the most likely diagnosis? 1- Healing fracture 2- Adamantinoma 3- Ewing’s sarcoma 4- Chondroblastoma 5- Eosinophilic granuloma
  • 134. Eosinophilic granuloma is a form of Langerhans Cell Histiocysosis (Histiocytosis X). It is a condition that mimics primary bone neoplasm. Presents with pain and swelling and any bone can be involved. Most frequently, a single bone is involved, but multiple bones can be affected. Radiographs will show a highly destructive lesion with well defined margins. Histology shows characteristic proliferating eosinophilic Langerhans cells, as well as multiple bilobed eosinophils with bright eosinophilic cytoplasm. It is commonly a self-limiting process, although low dose radiation, or curettage with bone grafting when articular surface is at risk or fracture is imminent are options. -Adamantinoma is a rare neoplasm of young adults found most frequently in the tibia, but the fibula, femur, ulna and radius are infrequently involved. Radiographs shows multiple “bubbly” sharply circumscribed lucent defects of different sizes, with sclerotic bone between. -Ewing’s Sarcoma is a small round cell sarcoma of children and young adults. Most commonly found in pelvis, distal femur, proximal tibia, femoral diaphysis, and proximal humerus. Xrays demontrate a large destructive lesion that may be purely lytic or have variable amounts of reactive new bone formation. May have characteristic “onion skin” appearance caused by periosteum lifted off in multiple layers. - Chondroblastoma is a benign cartilage tumor found in young patients with open physes. Most common locations are the distal femur, proximal tibia and proximal humerus. Radiographs show a central region of bone destruction separated from the medullary cavity by a rim of sclerotic bone. Histo shows chondroblasts and scattered multinucleated giant cells.
  • 135. References 275. Mirra JM. Bone Tumors. Philadelphia, PA: Lea and Febiger; 1989:1022-1045. Pitcher JD, Weber KL. Benign fibrous and histiocytic lesions. In: Schwartz HS, ed. Orthopaedic Knowledge Update: Musculoskeletal Tumors 2. Rosemont, IL: American Academy of Orthopaedic Surgeons; 2007:121-132.